0% found this document useful (0 votes)
279 views

MRCPUK Sample Part 1 PDF

The examinee received a score of 0/200 on an MRCP(UK) exam covering various clinical scenarios. The exam consisted of 9 multiple choice questions testing diagnosis and management of conditions like acute cerebellar hemorrhage, drug adverse effect profiles, chronic lymphocytic leukemia, IgA vasculitis, hepatitis B serology, antiphospholipid syndrome, cardiac arrhythmias on Holter monitoring, giardiasis, and HIV. The examinee did not provide answers for any of the questions.

Uploaded by

A.h.Murad
Copyright
© © All Rights Reserved
We take content rights seriously. If you suspect this is your content, claim it here.
Available Formats
Download as PDF, TXT or read online on Scribd
0% found this document useful (0 votes)
279 views

MRCPUK Sample Part 1 PDF

The examinee received a score of 0/200 on an MRCP(UK) exam covering various clinical scenarios. The exam consisted of 9 multiple choice questions testing diagnosis and management of conditions like acute cerebellar hemorrhage, drug adverse effect profiles, chronic lymphocytic leukemia, IgA vasculitis, hepatitis B serology, antiphospholipid syndrome, cardiac arrhythmias on Holter monitoring, giardiasis, and HIV. The examinee did not provide answers for any of the questions.

Uploaded by

A.h.Murad
Copyright
© © All Rights Reserved
We take content rights seriously. If you suspect this is your content, claim it here.
Available Formats
Download as PDF, TXT or read online on Scribd
You are on page 1/ 129

Membership of the Royal Colleges of Physicians of the United Kingdom

My MRCP(UK)

COVID-19 News About us MRCP(UK) examinations Get involved Contact us

Home  >  Exam  >  Exam results

Exam results
Exam completed, results below.

Your total score is 0/200


Results and explanations below

Question 1 No Answer Given Incorrect

A 65-year-old woman presented with a 12-hour history of sudden-onset gait unsteadiness, vomiting and
headache, followed by increasing drowsiness.

What is the most likely diagnosis?

Answers

A: acute cerebellar haemorrhage


B: acute subdural haemorrhage
C: frontal subdural empyema
D: herpes simplex encephalitis
E: pituitary apoplexy

Correct answer: A
Explanation
The sudden onset suggests a vascular cause and the rapid worsening with drowsiness suggests an expanding
space-occupying lesion with rising intracranial pressure and risk of herniation. The description is classic for a
cerebellar haemorrhage which, by its location in the posterior fossa, causes this clinical scenario. It is a
neurological emergency, requiring rapid identification.

Question 2 No Answer Given Incorrect


A post-marketing observational study of a new drug was conducted on 5000 patients following clinical trials.

What best describes the data generated from this type of study?

Answers

A: comparative efficacy
B: cost–benefit
C: cost effectiveness
D: potency
E: profile of adverse effects

Correct answer: E
Explanation
Post-authorisation safety studies (PASS) are observational (non-randomised) studies conducted after a new
drug is introduced into clinical use. Their aim is to characterise the adverse-effect profile, particularly in
patients not studied during clinical trials, such as those with liver and renal disease, pregnant women,
and children. 

Question 3 No Answer Given Incorrect

A 79-year-old woman was admitted for elective hip-replacement surgery.

On examination, she was pale. There was 2-cm splenomegaly and there were small discrete axillary lymph
nodes.

Investigations:

haemoglobin 107 g/L (115–165)


white cell count 34.5 × 109/L (4.0–11.0)
platelet count 183 × 109/L (150–400)
What is the most likely diagnosis?

Answers

A: acute myeloid leukaemia


B: chronic lymphocytic leukaemia
C: chronic myeloid leukaemia
D: myelodysplasia
E: myelofibrosis

Correct answer: B
Explanation
The presence of splenomegaly, anaemia and lymph nodes would make chronic lymphocytic leukaemia the
most likely possibility. Acute myeloid leukaemia is unlikely to be diagnosed as an incidental finding and
would more likely be associated with thrombocytopenia. Chronic myeloid leukaemia is a possibility, although
this is a less common diagnosis and an elevated platelet count would be expected. Myelodysplasia would not
usually be associated with a high white count. Myelofibrosis is a possible diagnosis, although the high white
count with normal platelet count makes this less likely. It would be expected to request a white cell
differential in the clinical setting.

Question 4 No Answer Given Incorrect

A 17-year-old boy presented with a non-blanching rash over his legs, a swollen knee and painless, visible
haematuria.

Urinalysis showed blood 3+, protein 1+.

Investigations:

serum creatinine 210 µmol/L (60–110)


   
urine culture negative
   
ultrasound scan of kidneys normal
What glomerular abnormality is most likely to be present at renal biopsy?

Answers

A: focal segmental sclerosis


B: foot process fusion
C: linear deposition of IgG on the basement membrane
D: mesangial deposition of IgA
E: thickening of basement membranes

Correct answer: D
Explanation
Typical presentation of IgA vasculitis (also known as Henoch–Schönlein purpura; this eponym is no longer
used in the exam) with a rash, arthritis and acute glomerulonephritis. The IgA is seen on
immunofluorescence.

Question 5 No Answer Given Incorrect

A 24-year-old woman from Zimbabwe attended the genitourinary medicine clinic for a sexual health screen.
She was asymptomatic and had been previously well.
Investigations:

anti-hepatitis B core IgG antibody positive


anti-hepatitis B core IgM antibody negative
anti-hepatitis B surface antibody positive
hepatitis B surface antigen negative
What is the most likely diagnosis?

Answers

A: acute hepatitis B
B: chronic hepatitis B
C: false-positive hepatitis B serology
D: previous infection with hepatitis B
E: previous vaccination for hepatitis B

Correct answer: D
Explanation
This a question addressing serological testing for hepatitis B infection. It is important to distinguish between
surface and core antibodies, and between antibody and antigen. Read the question carefully!

A   acute hepatitis B would be indicated by positive anti-hepatitis B core IgM antibody

B   chronic hepatitis B would be indicated by positive hepatitis B surface antigen 

C   false-positive hepatitis B serology would be indicated by positive anti-hepatitis B core IgG antibody alone

D   previous infection with hepatitis B is correct as indicated by positive anti-hepatitis B surface antibody
and anti-hepatitis B core IgG antibody with negative hepatitis B surface antigen

E   previous vaccination for hepatitis B would be indicated by positive anti-hepatitis B surface antibody but
negative anti-hepatitis B core IgG antibody

Question 6 No Answer Given Incorrect

A 45-year-old woman presented with right flank pain. She had a 4-year history of hypertension and
progressive cognitive impairment.

On examination, she had livedo reticularis and tenderness in the right flank. Her BP was 185/105 mmHg.
Urinalysis showed blood 3+, protein 1+.

Investigations:
haemoglobin 129 g/L (115–165)
white cell count 8.7 × 109/L (4.0–11.0)
platelet count 83 × 109/L (150–400)
   
serum creatinine 106 µmol/L (60–110)
What antibody test is most likely to be positive?

Answers

A: anticardiolipin
B: anticentromere
C: anti-glomerular basement membrane
D: antimitochondrial
E: anti-neutrophil cytoplasmic

Correct answer: A
Explanation
She has antiphospholipid syndrome and presented with probable renal vein thrombosis to account for her
flank pain. The other features are also typical of this condition. The other answer options are not typical of
this presentation.

Question 7 No Answer Given Incorrect

A 37-year-old woman presented with a history of intermittent lightheadedness.

Examination was normal.

Investigations:

ECG normal
   
atrial and ventricular premature beats; nocturnal
bradycardia and Mobitz type I atrioventricular block,
24-hour ambulatory ECG tracing
and supraventricular tachycardia

Which abnormality on the 24-hour ambulatory ECG is clinically most important?

Answers

A: atrial premature beats


B: profound sleep-associated bradycardia
C: supraventricular tachycardia
D: transient Mobitz type I AV block
E: ventricular premature beats

Correct answer: C
Explanation
Supraventricular tachycardia is the best answer. It would provide a plausible explanation for the patient’s
symptoms, and may merit specific treatment (either drugs or ablation).

Question 8 No Answer Given Incorrect

A 28-year-old man presented with a 1-month history of weight loss, abdominal distension, flatulence and
foul-smelling diarrhoea following a visit to India.

Investigations:

anti-tissue transglutaminase antibodies negative


   
stool cultures and microscopy negative
What is the most likely diagnosis?

Answers

A: acute HIV seroconversion illness


B: coeliac disease
C: giardiasis
D: hookworm infection (ancylostomiasis)
E: viral gastroenteritis

Correct answer: C
Explanation
The timing of the diarrhoeal symptoms and the travel history to an endemic area makes giardiasis the most
likely diagnosis. Stool cultures are often falsely negative in giardiasis. Coeliac disease is much less likely as
anti-tissue transglutaminase antibodies are negative. Viral gastroenteritis is unlikely to be so chronic. Acute
HIV and hookworm rarely present in such a fashion and are less plausible than giardiasis.

Question 9 No Answer Given Incorrect

A 30-year-old woman presented with a 6-month history of tremor and difficulty in speaking.

On examination, she was found to have increased muscle tone in all four limbs, bradykinesia and 4-cm
hepatomegaly.

What laboratory finding would best support a diagnosis of Wilson’s disease?


Answers

A: increased incorporation of radioactive copper into caeruloplasmin


B: low hepatic copper content
C: low serum caeruloplasmin concentration
D: low serum free copper concentration
E: low urinary copper concentration

Correct answer: C
Explanation
Wilson’s disease is an autosomal recessive disorder of copper metabolism. Copper accumulation in tissue
commonly presents with hepatic, neurological and psychiatric signs and symptoms. Levels of caeruloplasmin
are low in most, but not all, cases. Hepatic, serum and urinary copper levels are all elevated. Caeruloplasmin
is the molecule by which about 95% of copper is normally carried. In those affected by Wilson’s disease,
absence of ATP7B, also known as the Wilson’s disease protein, means caeruloplasmin cannot be correctly
synthesised in hepatocytes, resulting in an effective absence in serum.

Question 10 No Answer Given Incorrect

What dietary substrate is broken down into glucose and galactose by the action of intestinal enzymes?

Answers

A: fructose
B: lactose
C: maltose
D: mannose
E: sucrose

Correct answer: B
Explanation
Lactose is a disaccharide which contains galactose and glucose. Maltose is a disaccharide, composed of
glucose molecules. Glucose and fructose combine to form sucrose. Fructose is a monosaccharide and
mannose is an aldohexose monomer.

Question 11 No Answer Given Incorrect

A 16-year-old girl presented with non-scaly, discrete areas of hair loss on the scalp. She had a history of
atopic eczema and had a number of depigmented areas on her hands and around her eyes.

What is the most likely diagnosis?

Answers
A: alopecia areata
B: hypothyroidism
C: lupus erythematosus
D: seborrhoeic dermatitis
E: trichotillomania

Correct answer: A
Explanation
Alopecia areata (option A) is an immunologically based disorder where there are discrete areas of hair loss
but without scarring or scaling, and patients with this condition have a higher incidence of atopic eczema,
hence option A is correct. In lupus erythematosus, there is usually active inflammation with scarring, whereas
in trichotillomania, there are residual short hairs at the affected site, frequently of the same length, because
the patient generally finds it difficult to pull the shorter newly growing hairs. Hypothyroidism can give patchy
hair thinning with brittle, straw-like hair and bald patches, along with other symptoms of hypothyroidism
(such as weight gain and constipation). Seborrhoeic dermatitis generally gives diffuse scaling on the scalp
without hair loss; although it can cause some areas of hair loss, this would be associated with scaling of the
scalp.

Question 12 No Answer Given Incorrect

A 72-year-old man presented following an episode of collapse. There had been two similar episodes recently,
each lasting about 1 minute. He had had an anterior myocardial infarction 4 years previously.

On examination, he was orientated and symptom-free with a regular pulse of 80 beats/min. His BP was
140/80 mmHg and the apex beat was displaced to the left. There was an apical systolic murmur. There were
no signs of trauma.

Investigations:

sinus rhythm; anterior Q waves and anterior ST-segment elevation


ECG
without reciprocal depression
What is the most likely diagnosis?

Answers

A: acute anterior myocardial infarction


B: cerebral embolism
C: epilepsy
D: pulmonary embolism
E: ventricular tachycardia

Correct answer: E
Explanation
Ventricular tachycardia is the best answer. The description of collapse is not comprehensive, but there have
been several short-lasting, self-limiting episodes suggestive of cardiogenic syncope. The patient has a history
of myocardial infarction, clinical evidence of cardiomegaly, and an ECG typical of left ventricular aneurysm
(persistent ST elevation with Q waves). In this situation, ventricular tachyarrhythmia would be high on the
differential list. Bradyarrhythmia would be a plausible alternative, but is not given as an option here.

The history of recurrent collapse is not suggestive of acute myocardial infarction.

This patient would be at fairly high risk for cerebral embolism, but this does not commonly present with
isolated or recurrent syncope.

Epilepsy generally presents with more protracted loss of consciousness, and other features such as aura,
posture and tone abnormalities, tongue-biting, incontinence, and post-ictal confusion, which are not present
here.

Pulmonary embolus is another plausible, although less frequent, cause of collapse. However, there is no
mention of chest pain or dyspnoea, and there are no particular risk factors, so it would be a less likely cause
than arrhythmia in this case.

Question 13 No Answer Given Incorrect

A 26-year-old woman presented with unequal pupils.

On examination, the right pupil was larger than the left and did not react to light, directly or consensually.
On convergence, the right pupil reacted very slowly, but eventually became smaller than the left pupil.

What is the best description of this type of pupillary abnormality?

Answers

A: Adie’s tonic pupil


B: afferent pupillary defect
C: Argyll Robertson pupil
D: Horner’s syndrome
E: third cranial nerve palsy

Correct answer: A
Explanation
This is a typical description of an Adie’s pupil, often associated with reduced or absent deep tendon reflexes.
An afferent pupillary defect is due to impaired ipsilateral optic nerve function, causing brief paradoxical
pupillary dilatation to light with the “swinging torch” test. Argyll Robertson pupils accommodate but do not
react and are typically bilaterally small. A Horner’s would include ptosis with ipsilateral small pupil, and a third
cranial nerve palsy might include ipsilateral ptosis and pupillary enlargement.
Question 14 No Answer Given Incorrect

A 45-year-old man had recurrent nephrolithiasis. Renal function tests and serum calcium measurements were
normal.

Investigations:

24-h urinary calcium 8.8 mmol (2.5–7.5)


24-h urinary urate 3.0 mmol (<3.6)
24-h urinary oxalate 0.20 mmol (0.14–0.46)
24-h urinary citrate 0.2 mmol (0.3–3.4)
What is the most useful therapy to reduce stone formation?

Answers

A: allopurinol
B: bendroflumethiazide
C: dietary calcium restriction
D: penicillamine
E: potassium citrate

Correct answer: E
Explanation
The 24-h urinary calcium is marginally elevated and is not an indication for thiazide diuretic therapy. Urinary
citrate reduces urinary supersaturation of calcium salts by forming soluble complexes with calcium ions, and
by inhibiting crystal growth and aggregation, hence the use of potassium citrate in recurrent stone
formation. There is no data to warrant the use of allopurinol (for acute uric acid nephropathy) or
penicillamine. Low-calcium diets lead to increased oxalate absorption production, which is counterproductive.

Question 15 No Answer Given Incorrect

A 38-year-old woman required extraction of her wisdom teeth. She gave a history of haemorrhage after a
dental extraction 10 years previously, when she had required suturing. There had been no history of excessive
bleeding before this.

What is the most likely diagnosis?

Answers

A: factor V Leiden
B: factor IX deficiency
C: factor XII deficiency
D: primary antiphospholipid syndrome
E: von Willebrand disease

Correct answer: E
Explanation
The question stem implies that there is a longstanding tendency to bleed following dental extraction. The
answer options require the candidate to select the most likely disorder that could explain this bleeding.

Von Willebrand disease is the correct answer as this is a relatively common bleeding disorder that can
manifest in adults, as excessive bleeding after dental extractions or other minor surgical procedures. On
further questioning of female patients, there is often a history of menorrhagia. Factor V Leiden is not correct
as this is an inherited disorder that is associated with an increased risk of thrombosis, not bleeding. Factor IX
deficiency is an X-linked recessive disorder and would not be expected to affect females or a 38-year-old
patient. Factor XII deficiency is a rare autosomal recessive condition that is not associated with a bleeding
tendency in the majority of cases, although it is associated with a marked prolongation of the activated
partial thromboplastin time and may, incorrectly, be assumed to influence bleeding risk. Primary
antiphospholipid syndrome is associated with a tendency to thrombosis, not bleeding.

Question 16 No Answer Given Incorrect

A 30-year-old man presented with increasing breathlessness and wheeze. He smoked 5 cigarettes per day.

Investigations:

serum α1-antitrypsin 0.12 g/L (1.1–2.1)


   
FEV1 0.85 L (3.4–5.1)
FVC 4.75 L (4.2–6.5)
transfer factor for CO (TLCO) 4.1 mmol/min/kPa (7.1–12.7)
Which protease inhibitor phenotype is most likely to be present?

Answers

A: MM
B: MZ
C: SS
D: SZ
E: ZZ

Correct answer: E
Explanation
Very low serum α1-antitrypsin levels are associated with severe obstructive airways disease.
Question 17 No Answer Given Incorrect

A 35-year-old woman with chronic kidney disease stage V was being considered for erythropoietin therapy;
her eGFR was 12 mL/min/1.73 m2 (>60).

On examination, her BP was 140/90 mmHg and she had an arteriovenous fistula in situ for planned
haemodialysis.

Investigations:

haemoglobin 95 g/L (115–165)


MCV 84 fL (80–96)
What is the most likely outcome of erythropoietin therapy?

Answers

A: improved BP control
B: improved exercise tolerance
C: increased ventricular hypertrophy
D: reduced likelihood of red cell aplasia
E: stabilisation of renal function

Correct answer: B
Explanation
Erythropoietin therapy will stimulate bone marrow production of red blood cells and allow more oxygen
delivery to her tissues for better exercise tolerance. Erythropoietin has the potential to worsen BP control, but
studies have shown that its use is associated with reduction of left ventricular hypertrophy in patients with
advanced renal impairment and anaemia. It has no effect on renal function. There have been reports of
erythropoietin-induced red cell aplasia although this is now rare with the use of newer forms of
erythropoietin.

Question 18 No Answer Given Incorrect

A 54-year-old woman, who had undergone subtotal thyroidectomy for Graves’ disease 20 years previously,
presented for a review. She was well.

Investigations:

serum cholesterol 6.8 mmol/L (<5.2)


serum HDL cholesterol 1.10 mmol/L (>1.55)
   
serum thyroid-stimulating hormone 16.2 mU/L (0.4–5.0)
serum free T4 13.0 pmol/L (10.0–22.0)
What is the most appropriate initial treatment for her hyperlipidaemia?

Answers

A: bezafibrate
B: ezetimibe
C: levothyroxine
D: oestrogen replacement therapy
E: simvastatin

Correct answer: C
Explanation
This question is testing the candidate’s understanding of secondary causes of hypercholesterolaemia. The
history of a subtotal thyroidectomy would make thyroid dysfunction probable. Correcting an underactive
thyroid is important as it is likely to reduce cholesterol levels; also, starting cholesterol-lowering medication in
patients who are hypothyroid can increase the risk of adverse effects (myalgia) hence levothyroxine is the best
initial treatment.

Question 19 No Answer Given Incorrect

A 27-year-old woman presented with a right-sided thyroid swelling with associated cervical


lymphadenopathy.

What is the most likely cause?

Answers

A: anaplastic carcinoma
B: follicular adenoma
C: follicular carcinoma
D: Hashimoto’s thyroiditis
E: papillary carcinoma

Correct answer: E
Explanation
Malignancy must be considered in any patient with thyroid swelling and associated cervical
lymphadenopathy. Papillary thyroid carcinoma is the most common thyroid malignancy and, therefore, in the
absence of any further patient history or information in the question, is the correct answer.

Question 20 No Answer Given Incorrect


A 77-year-old man presented with increasing pains around his lower back and lower limb girdle. He had
recently presented with symptoms of hesitancy and post-micturition dribbling.

Investigations:

ESR 28 mm/1st h (<20)


   
serum corrected calcium 2.34 mmol/L (2.20–2.60)
serum phosphate 0.80 mmol/L (0.80–1.45)
serum alkaline phosphatase 2985 U/L (45–105)
   
serum prostate-specific antigen 6 μg/L (<4)
What is the most likely cause of this man’s pain?

Answers

A: insufficiency fracture of the pelvis


B: osteomalacia
C: Paget’s disease of bone
D: polymyalgia rheumatica
E: prostatic carcinoma with metastases

Correct answer: C
Explanation
The history of localised bony pain with significantly elevated serum alkaline phosphatase and normal serum
corrected calcium and serum phosphate in an elderly man is entirely consistent with a diagnosis of Paget's
disease.

The history is not suggestive of insufficiency fracture of the pelvis. In osteomalacia, you would expect to see
decreased serum corrected calcium and serum phosphate alongside the raised serum alkaline phosphatase.
While he has pain around his pelvic girdle, the ESR is normal for his age and elevated serum alkaline
phosphatase would make polymyalgia rheumatica unlikely. In advanced metastatic prostate cancer, you
would expect to see a greater elevation in serum prostate-specific antigen and raised serum corrected
calcium and phosphate. Lower urinary tract symptoms of hesitancy and post-micturition dribbling are very
common in this age group.

Question 21 No Answer Given Incorrect

A 50-year-old man presented with a 2-day history of multiple painful joints. Two weeks previously, he had
been started on treatment with allopurinol for tophaceous gout. His only other medication was paracetamol.
He had a history of excess alcohol intake.
On examination, his temperature was 37.5°C and there was acute inflammation of the finger joints, wrists,
knees and ankles.

Investigations:

serum gamma glutamyl transferase 90 U/L (<50)


serum CRP 180 mg/L (<10)
serum urate 0.65 mmol/L (0.23–0.46)
What is the most likely cause of his symptoms?

Answers

A: acute pseudogout
B: alcoholic binge
C: allergic reaction to allopurinol
D: allopurinol therapy
E: joint sepsis

Correct answer: D
Explanation
The stem gives information that this man has established gout. One could argue that gout treatment should
have been started before tophi appeared, but in real life it is not unusual. Urate-lowering therapy has a high
chance of precipitating acute gout attacks. The scenario here exemplifies this common phenomenon a
clinician to be aware of. Joint sepsis is always a differential diagnosis for acute gout. Here, his temperature is
not high enough for polyarticular sepsis, which in any case is not very common; serum CRP elevation also
does not favour sepsis. Predisposing factors are very common in polyarticular joint sepsis, which again points
away from sepsis. It is a relevant topic and tests candidates’ knowledge around urate-lowering therapy
initiation.

Question 22 No Answer Given Incorrect

A 17-year-old boy presented with breathlessness and night sweats.

Investigations confirmed a diagnosis of Burkitt lymphoma.

What is the most likely underlying abnormality of gene expression?

Answers

A: expression of BCR-ABL
B: loss of p53
C: overexpression of BCL-2
D: overexpression of c-Myc
E: overexpression of JAK2

Correct answer: D
Explanation
Burkitt lymphoma is an aggressive non-Hodgkin lymphoma characterised by the overexpression of c-Myc,
most usually due to a chromosomal translocation between chromosomes 8 and 14, bringing the c-Myc gene
under the control of the immunoglobulin heavy chain promoter and thus rendering it constitutively active.
BCR-ABL is associated with chronic myeloid leukaemia, TP53 mutations are associated with various different
cancer types, overexpression of BCL2 is associated with B-cell non-Hodgkin lymphoma although not
specifically with Burkitt lymphoma, and JAK2 mutations are associated typically with myeloproliferative
disease in particular primary polycythaemia.

Question 23 No Answer Given Incorrect

A 37-year-old woman with immune thrombocytopenia failed to respond to corticosteroid therapy.


Splenectomy was planned.

What is the optimum time for pneumococcal vaccination?

Answers

A: 1 month after surgery


B: 1 month before surgery
C: 1 week after surgery
D: 1 week before surgery
E: perioperatively

Correct answer: B
Explanation
At least 2 weeks before surgery is recommended by the British Committee for Standards in Haematology
(BCSH).

Question 24 No Answer Given Incorrect

A 30-year-old man had moved recently to the UK. He informed his doctor that he had maturity-onset
diabetes of the young.

What feature, if present, most strongly suggests that this is the correct diagnosis?

Answers

A: a BMI of 30.0 kg/m2 (18.0–25.0)


B: a poor response to glipizide therapy
C: a strong family history of diabetes mellitus
D: recurrent episodes of diabetic ketoacidosis
E: X-linked recessive inheritance

Correct answer: C
Explanation
Maturity onset diabetes of the young is suspected when diabetes mellitus occurs in a young person with a
strong family history of diabetes mellitus.

Question 25 No Answer Given Incorrect

A 17-year-old girl presented with a single, painless, enlarged cervical lymph node. She was asymptomatic.

Investigations:

chest X-ray enlarged mediastinal lymph nodes


What is the most likely diagnosis?

Answers

A: angioimmunoblastic T-cell lymphoma


B: extramedullary plasmacytoma
C: follicular B-cell non-Hodgkin lymphoma
D: Hodgkin lymphoma
E: mantle cell lymphoma

Correct answer: D
Explanation
The stem makes clear that this adolescent female has cervical and mediastinal lymphadenopathy. She has no
other described symptoms and so it should be presumed that she is otherwise well. The presence of
mediastinal lymphadenopathy should be considered pathological.

The most common single malignant diagnosis in a patient of this age is Hodgkin lymphoma. This condition
most commonly presents with neck and mediastinal lymphadenopathy and is a condition with a peak in
incidence during adolescence. Patients may be asymptomatic.

Angioimmunoblastic T-cell lymphoma is uncommon in this age group and would classically present with more
widespread lymphadenopathy and a rash.

Extramedullary plasmacytoma would be unlikely in this age group and would classically present with lesions
in the upper respiratory tract, such as the oropharynx.

Follicular B-cell non-Hodgkin lymphoma would be much less likely than Hodgkin lymphoma in this age
group, although it could present with asymptomatic lymphadenopathy.
Mantle cell lymphoma is unlikely in this age group and would be expected to present with more widespread
lymphadenopathy and splenomegaly.

Question 26 No Answer Given Incorrect

A 70-year-old man presented accompanied by his wife, who stated that her husband had lost all interest in
life.

What would support a diagnosis of dementia rather than depressive disorder?

Answers

A: agitation
B: complaint of poor memory
C: impaired short-term memory
D: loss of libido
E: urinary incontinence

Correct answer: E
Explanation
A patient making a subjective compliant of poor memory would be more common in depression. An
objective finding of poor short-term memory is more common in dementia.

Agitation can occur in both.

Loss of libido is common in depression but not dementia.

Urinary incontinence is very rare in depression and occurs in dementia especially as it advances, and is caused
by normal pressure hydrocephalus

Question 27 No Answer Given Incorrect

A 48-year-old woman was admitted to the emergency department with a 2-hour history of haemoptysis. She
had been treated for pulmonary tuberculosis when she was 30 years old and had developed a right upper
lobe cavity containing an aspergilloma. She continued to cough up large amounts of blood over the next
24 hours and the decision was made to perform a radiologically guided embolisation.

Which blood vessel is most likely to be the target for embolisation?

Answers

A: brachiocephalic artery
B: bronchial artery
C: intercostal artery
D: internal thoracic artery
E: pulmonary artery

Correct answer: B
Explanation
The most frequent diseases causing haemoptysis are bronchiectasis, tuberculosis, fungal infections and
cancer. Causes of bleeding from the large vessels include infectious, cardiovascular, congenital, neoplastic
and vasculitic diseases. In cases of severe haemoptysis requiring treatment, the source of bleeding originates
from bronchial and pulmonary arteries in 90% and 5% of cases, respectively.

Question 28 No Answer Given Incorrect

A 32-year-old woman returned from a holiday in the Mediterranean with a suntan and numerous
hypopigmented, slightly scaly lesions on the neck and upper trunk. She mentioned that the same areas had
been fawn-coloured in the months before her holiday and that these areas of skin had failed to tan during
her time in the Mediterranean.

What is the most likely diagnosis?

Answers

A: chronic plaque psoriasis


B: discoid eczema
C: pityriasis rosea
D: pityriasis versicolor
E: seborrhoeic dermatitis

Correct answer: D
Explanation
The correct answer is pityriasis versicolor, which typically affects the neck and upper trunk, causing beige- or
fawn-coloured areas on untanned skin but these areas become hypopigmented while the rest of the patient’s
skin becomes tanned following repeated sun exposure. Sun exposure can improve some other skin diseases
such as psoriasis and eczema, and this may result in hypopigmented areas at the site of the previous lesions
in some cases, but one would expect a history of lesions at other sites, for example on the upper and lower
limbs, as well as on the trunk. Seborrhoeic dermatitis is a common cause of dandruff and consists of an
erythematous, scaly, macular rash affecting the scalp, eyebrows, nasolabial folds and, less frequently, the
centre of the chest (presternal region), axillae and perianal area. Pityriasis rosea affects the trunk and limbs,
and starts with a “herald patch” (a single patch) followed 1–2 weeks later by more generalised erythematous
scaly patches on the trunk and upper limbs, which slowly spread downwards to the thighs and subsequently
fade over the next 6–8 weeks.

Question 29 No Answer Given Incorrect


A 65-year-old woman developed bilateral lower limb swelling. She had been taking regular ibuprofen for
joint pain for the previous 2 weeks. She had a past medical history of urinary tract infections.

On examination, her BP was 139/98 mmHg and she had pitting oedema below the knees. Urinalysis showed
protein 2+, red blood cells 1+, leukocytes 3+ and no nitrites.

Investigations:

serum creatinine 458 µmol/L (60–110)


   
urine protein:creatinine ratio 130 mg/mmol (<30)
What is the most likely cause of the renal impairment?

Answers

A: acute tubular necrosis


B: IgA nephropathy
C: membranous glomerulonephropathy
D: papillary necrosis
E: tubulointerstitial nephritis

Correct answer: E
Explanation
She has acute kidney injury with modest proteinuria and leukocyturia which is in keeping with interstitial
nephritis. NSAIDs are associated with interstitial nephritis. Acute tubular necrosis is classically associated with
a bland urinalysis. IgA nephropathy is not associated with NSAID use and is therefore less likely. Membranous
glomerulonephropathy is more commonly associated with nephrotic range proteinuria (urine PCR
>300mg/mmol). Papillary necrosis can be associated with NSAID use but also with flank pain and higher
levels of haematuria. A renal biopsy would be required to differentiate between the possible causes for her
renal impairment but tubulointerstitial nephritis is the most likely diagnosis based on the information given.

Question 30 No Answer Given Incorrect

A 47-year-old man presented with a 6-month history of episodic sweating and hunger. He had gained 10 kg
in weight and drank 10 units of alcohol per week.

Investigations:

full blood count normal


   
fasting plasma glucose 4.0 mmol/L (3.0–6.0)
   
liver function tests normal
What is the most appropriate next investigation?

Answers

A: CT scan of pancreas
B: EEG
C: home blood glucose monitoring
D: plasma glucose concentration following a prolonged (72-h) fast
E: serum C-peptide concentration

Correct answer: D
Explanation
Recurrent sweating and hunger in the presence of weight gain and a borderline low fasting glucose profile
prompts the consideration for insulinoma. Measuring prolonged (72-h) fasting glucose levels is the gold
standard initial test for insulinoma.

CT scan is not an appropraite initial investigation without proving true hypoglycaemia and might miss an
insulinoma. There is not enough to suggest that the patient needs investigation for epilepsy. Home glucose
monitoring will not aid the diagnosis. Serum C-peptide is not beneficial in the absence of true
hypoglycaemia.

Question 31 No Answer Given Incorrect

A 25-year-old woman presented complaining of having loose stools five to six times per day. One year
previously, she had undergone resection of her terminal ileum for Crohn’s disease. Her only drug therapy was
mesalazine.

On examination, there was no abdominal tenderness.

Investigations:

haemoglobin 126 g/L (115–165)


   
serum albumin 38 g/L (37–49)
serum CRP 4 mg/L (<10)
What is the most likely diagnosis?

Answers

A: active Crohn’s disease


B: adverse effect of mesalazine
C: bile salt-induced diarrhoea
D: enteric infection
E: irritable bowel syndrome

Correct answer: C
Explanation
Bile salt-induced diarrhoea (or malabsorption) is a common and recognised consequence of terminal ileal
resection as bile salts are reabsorbed in the terminal ileum. Loss of the terminal ileum leads to excess bile salts
in the colon which can act as an irritant, causing diarrhoea. Active Crohn’s disease is less likely as the serum
CRP and haemoglobin are normal. Adverse effects of mesalazine tend to occur soon after initial prescription.
Enteric infection is less likely from the chronicity of symptoms given. While irritable bowel syndrome is a
possible orrect answer and is recognised to occur in inflammatory bowel disease, bile salt remains more likely
in the context of a terminal ileal resection and needs to be excluded first.

Question 32 No Answer Given Incorrect

A 71-year-old woman required a transfusion of 2 units of blood after a hip replacement. One week later, her
haemoglobin concentration had fallen by 42 g/L.

What associated finding is most likely to indicate a delayed transfusion reaction?

Answers

A: conjugated hyperbilirubinaemia
B: elevated D-dimer
C: haemoglobinuria
D: haemosiderinuria
E: positive direct antiglobulin test

Correct answer: E
Explanation
A delayed haemolytic transfusion, by definition, occurs more than 24 hours after transfusion but is usually
clinically evident approximately 2 weeks after transfusion. It is caused by boosting of a previously generated
alloantibody and causes a positive direct antiglobulin test, a raised unconjugated bilirubin with clinical
jaundice, anaemia, reticulocytosis and a raised lactase dehydrogenase. The blood transfusion antibody screen
is positive and spherocytosis is seen on examination of the blood film. An unexplained fever typically occurs.
Haemoglobinuria would indicate a more significant acute haemolytic transfusion reaction (this is delayed, and
more benign) and haemosiderinuria a more chronic haemolytic condition.

Question 33 No Answer Given Incorrect

A 30-year-old woman presented with a 4-hour history of palpitations. She had no significant medical history.

Investigations:
ECG atrial fibrillation with a ventricular rate of 130 beats/min
What drug is most likely to restore sinus rhythm?

Answers

A: adenosine
B: bisoprolol
C: esmolol
D: flecainide
E: verapamil

Correct answer: D
Explanation
Flecainide is the best answer. Of the drugs listed, it is the one most likely to restore sinus rhythm in a young
patient with acute-onset atrial fibrillation. It is particularly indicated in patients who have accessory pathways.
Care has to be taken, however, to avoid using the drug in patients with left ventricular dysfunction or
ischaemic heart disease as it is negatively inotropic.

There is no role for adenosine in the treatment or diagnosis of atrial fibrillation. Bisoprolol, esmolol, and
verapamil might all restore sinus rhythm, but are less efficacious for this indication than flecainide.

Question 34 No Answer Given Incorrect

A 20-year-old woman presented with a 2-month history of an intensely pruritic rash on her trunk and limbs.
She was otherwise well.

On examination, she had multiple, violaceous, flat-topped papules over the flexor surfaces of the wrists, and
on the ankles and lower back. The papules were aggregated in a linear fashion at one site on her left
forearm.

What clinical feature is most likely to be present?

Answers

A: asymmetric oligoarthritis
B: burrows in finger webs
C: conjunctival scarring
D: involvement of buccal mucosa
E: non-scarring alopecia

Correct answer: D
Explanation
Lichen planus causes violaceous, flat-topped papules, which are often seen on the flexor surfaces of the
wrists, lower back and ankles as well as elsewhere on the skin. Lichen planus can koebnerise, which is when
the lesions arise at an area of skin that has been traumatised, and the linear pattern on left forearm would be
consistent with koebnerisation from scratching. Lichen planus frequently causes a lacy, white pattern on the
buccal mucosa, hence involvement of the buccal mucosa is the orrect answer. When lichen planus affects the
scalp (i.e. lichen planopilaris), it causes a scarring alopecia, so non-scarring alopecia is unlikely. Conjunctival
scarring with lichen planus has been reported but this is uncommon, so is incorrect. Burrows in finger webs
are seen in scabies, not lichen planus, so this option is incorrect. Lichen planus does not affect the joints, so is
incorrect.

Question 35 No Answer Given Incorrect

A 60-year-old woman with rheumatoid arthritis presented with an acutely inflamed right knee. The joint was
aspirated and methylprednisolone 80 mg was injected after the removal of 20 mL of non-purulent synovial
fluid. The knee improved following this procedure but her symptoms recurred and became much worse a
week later.

Investigations:

haemoglobin 105 g/L (115–165)


15.5 × 109/L (4.0–
white cell count
11.0)
13.5 × 109/L (1.5–
neutrophil count
7.0)
ESR 55 mm/1st h (<30)
What is the most appropriate next investigation?

Answers

A: blood cultures
B: serum CRP
C: serum urate
D: synovial fluid culture
E: X-ray of right knee

Correct answer: A
Explanation
The scenario in question is typical for septic arthritis following joint injection, which is a known complication
of the procedure, with an estimated prevalence of 4 cases per 10 000 injections. The most appropriate next
step, following new British Society for Rheumatology guidelines, is blood cultures, before aspiration and
antibiotics. The yield from blood culture is higher than from synovial fluid culture, and not all acute medical
trainees are competent in aspirating a joint. Hence, after taking blood culture, all patients with suspected
joint sepsis should be started on intravenous antibiotics. Joint aspiration and synovial fluid culture can follow.
Serum CRP is a non-specific marker of inflammation and could be raised in a flare of rheumatoid arthritis, or
for numerous other reasons. Crystal arthropathy is a well-recognised cause of monoarthritis but the clinical
history makes this less likely and the urate level can be normal in up to 25% of patients during an acute
attack so would not exclude the diagnosis. Finally, X-ray of the knee will most likely be normal in the acute
phase of septic arthritis or may show old changes, but cannot be used to confirm the cause of the swollen
joint.

Question 36 No Answer Given Incorrect

A 75-year-old man presented with weight loss, lethargy and repeated haemoptysis. He had been treated for
pulmonary tuberculosis 10 years previously.

Investigations:

serum IgE for Aspergillus positive


serum IgG for Aspergillus positive
   
chest X-ray a solid lesion at the left lung apex
What is the most likely diagnosis?

Answers

A: allergic bronchopulmonary aspergillosis


B: aspergilloma
C: bronchial carcinoma
D: invasive aspergillosis
E: reactivation of tuberculosis

Correct answer: B
Explanation
The differential diagnosis for this presentation includes all of the conditions listed. The positive serum IgE
and IgG for Aspergillus, and a solid lesion on chest X-ray strongly suggest an aspergilloma. Invasive
aspergillosis will not present with a solid mass and only occurs in someone who is immunosuppressed.
Patients with allergic bronchopulmonary aspergillosis have proximal bronchiectasis and present with
productive cough and wheeze but not haemoptysis.

Question 37 No Answer Given Incorrect

A 24-year-old man presented with agitation and central chest pain. He had been taking cocaine earlier that
day.

On examination, he had a pulse of 100 beats/min and a BP of 170/110 mmHg.


What complication is most likely?

Answers

A: hypercalcaemia
B: hyperkalaemia
C: hyperthermia
D: hyponatraemia
E: hypothermia

Correct answer: C
Explanation
Cocaine is a sympathomimetic drug that causes chest pain, hypertension, agitation and arrhythmias.

Hyperthermia and rhabdomyolysis are recognised complications resulting from its serotoninergic effects.

Question 38 No Answer Given Incorrect

A 51-year-old woman presented with a 6-month history of intermittent episodes of arthralgia, rash and
fevers. She complained of increasing fatigue, breathlessness and swollen ankles over the previous 2 months.

On examination, she had a purpuric rash on the lower extremities and a right-sided pleural effusion.
Urinalysis showed blood 1+, protein 3+.

Investigations:

ESR 140 mm/1st h (<30)


   
serum creatinine 140 µmol/L (60–110)
serum complement C4 <5 mg/dL (15–50)
   
1:600 dilution (negative at
antinuclear antibodies
1:20)
rheumatoid factor 90 kIU/L (<30)
What is the most likely diagnosis?

Answers

A: IgA vascultis
B: microscopic polyangiitis
C: mixed cryoglobulinaemia
D: rheumatoid arthritis
E: systemic lupus erythematosus
Correct answer: E
Explanation
The stem describes multisystem involvement. Investigations show a multisystem inflammatory complement-
consuming illness. Systemic lupus erythematosus is the most likely cause for this clinical presentation: it has
musculoskeletal, renal and skin symptomatology and systemic manifestation. IgA vasculitis is not common in
this age group and pleural involvement is extremely rare. Microscopic polyangiitis would affect the lung
parenchyma and not pleura. Mixed cryoglobulinaemia is rare and there are no predisposing illnesses here.
Rheumatoid arthritis (RA) is not a complement-consuming condition and would not cause renal involvement
at such an early stage; rheumatoid factor is not a diagnostic test or a specific test for RA.

Question 39 No Answer Given Incorrect

A 64-year-old man presented with increasing pain in his right elbow and tingling in the medial aspect of his
right hand. His job involved using a sledgehammer.

Examination was normal.

What is the most likely diagnosis?

Answers

A: C6 radiculopathy
B: compartment syndrome of the forearm
C: lateral epicondylitis
D: median nerve entrapment
E: ulnar nerve entrapment

Correct answer: E
Explanation
The distribution of the tingling would suggest an ulnar nerve lesion. The ulnar nerve is commonly entrapped
at the elbow (medial epicondyle) and can be exacerbated by repetitive or prolonged use of the elbow, as in
this man.

Question 40 No Answer Given Incorrect

A 75-year-old woman presented with a 6-month history of an ulcer over the right ankle. She had a history of
right deep venous thrombosis 5 years previously.

On examination, she had a superficial sloughing ulcer, 6 cm in diameter, over the medial malleolus.

What is the most appropriate investigation?

Answers
A: ankle–brachial pressure index
B: bacteriological swab of the ulcer
C: bilateral lower limb arteriography
D: right leg venography
E: venous duplex ultrasound scan

Correct answer: A
Explanation
This is a case of venous ulceration, and the information that indicates this is:

(i) the history of a deep venous thrombosis of the same leg some years previously, which will have damaged
the valves in the veins,
(ii) the site of the ulcer, over the ankle/medial malleolus, and
(iii) the superficial nature of the ulcer.

Treatment for venous ulceration is with a compression bandage, which allows the leg muscles to pump the
blood back to the heart when the patient walks because contracting the calf muscles presses against the
compression bandage and squeezes the blood up along the vein. However, compression is contraindicated if
there is insufficient arterial supply to the foot, because, in that case, the compression could lead to lower
limb ischaemia, so it is important to check blood flow into the leg before prescribing compression bandages.
Although one can feel for a pulse (dorsalis pedis or posterior tibialis), this is not as accurate as doing an
ankle–brachial pressure index (option A), which is therefore the correct answer.
 

Question 41 No Answer Given Incorrect

A 50-year-old woman presented with a 6-month history of diarrhoea and weight loss of 5 kg. She described
her stools as pale and bulky. She had a history of Raynaud’s phenomenon.

On examination, the skin over her fingers was shiny and tight.

What is the most likely cause of her diarrhoea?

Answers

A: bacterial overgrowth
B: gluten sensitivity
C: lactose intolerance
D: microscopic colitis
E: pancreatic insufficiency

Correct answer: A
Explanation
This patient has Raynaud’s phenomenon and skin changes consistent with systemic sclerosis. She has
developed small intestinal bacterial overgrowth, which is a common complication in systemic sclerosis,
occurring in up to 40% of patients. Systemic sclerosis affects the small intestine, causing reduced peristalsis
and small bowel dilatation, resulting in alterations in the small bowel bacterial flora. Features include loose
stools, steatorrhoea, malabsorption, abdominal discomfort and flatulence.

The other options would not be expected to be associated with skin thickening and Raynaud’s phenomenon.

Question 42 No Answer Given Incorrect

A 55-year-old man presented with a 12-month history of increasing breathlessness.

On examination, he had finger clubbing, and widespread end-inspiratory fine crackles on chest auscultation.

Investigations:

chest X-ray diffuse lung shadowing


What pulmonary function test result would be most characteristic of a diagnosis of idiopathic pulmonary
fibrosis?

Answers

A: FEV1/FVC ratio 0.6


B: peak flow 50% of predicted value
C: residual volume 110% of predicted value
D: total lung capacity 95% of predicted value
E: transfer factor for CO 50% of predicted value

Correct answer: E
Explanation
This patient has pulmonary fibrosis, probably usual interstitial pneumonitis (UIP). The clinical presentation is
typical. The most important and useful measurement is the diffusing capacity/transfer factor which is a
measure of how effectively oxygen diffuses from the alveolus into the bloodstream. Transfer factor for CO is
used to determine the severity of the fibrosis and can be used to monitor progression of disease. The other
measurements are less useful.

Question 43 No Answer Given Incorrect

A 42-year-old man was found to have abnormal liver function tests. He had had ulcerative colitis for 15
years. He had been treated with mesalazine but this had been stopped 8 years previously when his disease
had gone into remission.
Investigations:

serum albumin 40 g/L (37–49)


serum total bilirubin 15 µmol/L (1–22)
serum alanine aminotransferase 63 U/L (5–35)
serum aspartate aminotransferase 41 U/L (1–31)
serum alkaline phosphatase 741 U/L (45–105)
serum gamma glutamyl transferase 221 U/L (<50)
What is the most likely cause of these blood results?

Answers

A: autoimmune hepatitis
B: fatty liver
C: liver metastases
D: primary biliary cirrhosis
E: primary sclerosing cholangitis

Correct answer: E
Explanation
The pattern of derangement of liver function tests is suggestive of a cholestatic disorder such as primary
sclerosing cholangitis, which is recognised to occur in up to 10% of ulcerative colitis patients. Primary biliary
cirrhosis is not directly linked with ulcerative colitis; autoimmune hepatitis is a possibility but the pattern of
liver function tests is less suggestive and there is limited association with ulcerative colitis. Fatty liver is
possible but the pattern of liver function tests makes it less likely. Metastases are also possible but there is
nothing to suggest that in the history and it is less likely than primary sclerosing cholangitis in a 42-year-old
man.

Question 44 No Answer Given Incorrect

A 65-year-old man presented with gradually worsening exertional angina, and underwent percutaneous
coronary intervention with a drug-eluting stent. He was advised to take clopidogrel in addition to aspirin,
atenolol and simvastatin.

What is the predominant mechanism by which clopidogrel inhibits platelet aggregation?

Answers

A: it enhances the effect of circulating antithrombin


B: it inhibits binding of adenosine diphosphate
C: it inhibits cyclo-oxygenase-1
D: it irreversibly binds glycoprotein IIb/IIIa receptor sites
E: it prevents production of thromboxane A2
Correct answer: B
Explanation
Option B (it inhibits binding of adenosine diphosphate) is the best answer. Clopidogrel irreversibly inhibits
platelet aggregation via P2Y12 class ADP receptors.

Heparin enhances the activity of circulating antithrombin.

Aspirin inhibits production of thromboxane A2, through COX-1 inhibition; aspirin is a weakly selective COX-1
inhibitor.

Several drugs bind glycoprotein IIb/IIIa receptor sites on platelets, thereby preventing aggregation – abciximab
has high affinity.

Question 45 No Answer Given Incorrect

A 45-year-old man presented with recurrent epistaxis.

On examination, there were telangiectases on his lips and in his mouth.

A diagnosis of hereditary haemorrhagic telangiectasia was made.

What is the most likely mode of inheritance?

Answers

A: autosomal dominant
B: autosomal recessive
C: mitochondrial inheritance
D: sporadic mutation
E: X-linked recessive

Correct answer: A
Explanation
This condition is also known as Osler–Weber–Rendu syndrome and is inherited in an autosomal dominant
fashion.

Question 46 No Answer Given Incorrect

A 50-year-old man presented with a 24-hour history of agitation, confusion and suicidal thoughts. He had
started VAD chemotherapy (vincristine, doxorubicin and dexamethasone) 4 days previously, following a
diagnosis of multiple myeloma.
What is the most likely cause of his symptoms?

Answers

A: adjustment disorder
B: confusion secondary to sepsis
C: corticosteroid psychosis
D: reactive depression
E: vincristine encephalopathy

Correct answer: C
Explanation
Abrupt onset after chemotherapy and the finding of confusion would be unusual for adjustment disorder
and depression. Vincristine causes encephalopathy if given intrathecally by mistake. Sepsis is a possibility but
there is nothing else to suggest this in the stem. The finding of agitation and use of dexamethasone indicate
that corticosteroid psychosis is most likely and it is relatively common clinically.

Question 47 No Answer Given Incorrect

A 45-year-old man presented with a 1-month history of foot drop.

What feature most strongly favours an L5 radiculopathy over a peroneal nerve palsy?

Answers

A: complete paralysis of extensor hallucis longus


B: loss of sensation confined to the lower calf
C: positive Tinel’s sign lateral to the knee
D: weakness of ankle eversion
E: weakness of ankle inversion

Correct answer: E
Explanation
Weakness of inversion is not seen in common peroneal nerve lesion but may be present with L5 lesion
(tibialis posterior muscle is supplied by tibial nerve, L4/5). Tinel’s is a weak sign with low positive predictive
value.

Question 48 No Answer Given Incorrect

A 21-year-old man presented after a truamatic hemisection of his spinal cord at T10 level.

What is most likely to be present below the level of the lesion 2 months after the injury?
Answers

A: contralateral gross wasting


B: contralateral loss of pain and temperature sensation
C: contralateral loss of proprioception
D: contralateral upper motor neurone weakness
E: ipsilateral gross wasting

Correct answer: B
Explanation
Answering this question requires understanding of spinal cord anatomy. Since pain and temperature fibres
(spinothalamic) cross at or above the level they enter the spinal cord, hemisection of the cord causes
contralateral loss of pain and temperature sensation below the lesion. This is a feature of the Brown–Séquard
syndrome. It is too early to see disuse atrophy. Weakness and proprioception would be ipsilateral, not
contralateral features.

Question 49 No Answer Given Incorrect

A healthy 19-year-old medical student was asked to perform a Valsalva manoeuvre (forced expiration against
a closed glottis) for demonstration purposes in a physiology class.

What is the most likely initial haemodynamic response?

Answers

A: decreased JVP
B: decreased systolic BP
C: decreased venous return to the heart
D: increased cardiac output
E: increased pulse

Correct answer: C
Explanation
Increased intrathoracic pressure shifts blood from the pulmonary circulation to the left atrium, and also
causes reduced systemic return to the right side of the heart. JVP will increase initially and cardiac output
drops. Heart rate, and therefore pulse, drops during phase 1 of Valsalva manoeuvre.

Question 50 No Answer Given Incorrect

A 47-year-old woman was being treated with lithium for bipolar affective disorder.

On examination, her BP was 168/104 mmHg.


What is the most appropriate antihypertensive drug for her?

Answers

A: amlodipine
B: bendroflumethiazide
C: doxazosin
D: losartan
E: ramipril

Correct answer: A
Explanation
Although NICE guidelines recommend an ACE inhibitor as first-line treatment in patients under 55 years old,
amlodipine is the preferred choice as it does not cause a significant increase in serum lithium concentration.
ACE inhibitors, angiotensin II receptor antagonists and thiazide diuretics can all cause lithium toxicity by
reducing renal lithium clearance.

Question 51 No Answer Given Incorrect

A 43-year-old woman presented with a 1-week history of polyuria and nocturia without weight loss.

Clinical examination was normal.

Investigations:

serum sodium 142 mmol/L (137–144)


serum potassium 3.5 mmol/L (3.5–4.9)
serum corrected calcium 2.60 mmol/L (2.20–2.60)
random plasma glucose 7.0 mmol/L
serum osmolality 310 mosmol/kg (278–300)
   
urine osmolality 200 mosmol/kg (100–1000)
What is the most likely diagnosis?

Answers

A: compulsive polydipsia
B: diabetes insipidus
C: diabetes mellitus
D: hyperparathyroidism
E: syndrome of inappropriate antidiuretic hormone

Correct answer: B
Explanation
Raised serum osmolality with urine osmolality on the lower side in the setting of this patient’s symptoms
indicates diabetes insipidus. There is not enough in the question to suggest compulsive polydipsia; the
random plasma glucose level is not consistent with diabetes mellitus; calcium is not elevated which rules out
hyperparathyroidism; and syndrome of inappropriate antidiuretic hormone (SIADH) results in a low serum
osmolality.

Question 52 No Answer Given Incorrect

A 24-year-old woman presented with diarrhoea of 8 weeks’ duration. In the past week, she had noticed
blood in the stool, and was passing 10–12 stools per day and waking at night with the urge to defaecate.

On examination, she had a temperature of 38.0°C. Her abdomen was generally tender, without guarding or
rigidity.

What is the most appropriate next investigation?

Answers

A: colonoscopy
B: CT scan of abdomen
C: plain X-ray of abdomen
D: rigid sigmoidoscopy
E: stool microscopy

Correct answer: C
Explanation
It is critical to exclude toxic colon dilatation, as the likely diagnosis is acute severe ulcerative colitis. While
colonoscopy and rigid sigmoidoscopy will help make the diagnosis, they cannot exclude toxic dilatation,
which is a potential surgical emergency. Stool microscopy is unlikely to be helpful as the symptoms are
chronic and, although indicated, the results will take a few days to be available. A CT scan is not indicated
before a plain film in acute ulcerative colitis.

Question 53 No Answer Given Incorrect

In a large prospective trial of a statin in patients with angina, 1000 patients were treated with a statin and
1000 with a placebo. A total of 150 patients taking the placebo experienced a cardiovascular event,
compared with 100 taking the active drug.

  placebo statin total


cardiovascular event 150 100 250
no cardiovascular event 850 900 1750
What is the number-needed-to-treat (NNT) to prevent one cardiovascular event?

Answers

A: 2.5 (250/100)
B: 5 (250/50)
C: 10 (1000/100)
D: 20 (1000/50)
E: 40 (2000/50)

Correct answer: D
Explanation
50 extra events occurred in the placebo arm of 1000 people, so 1000 must be treated to prevent 50 events.
Therefore, 20 people are treated for every 1 event prevented, or NNT=20.

Question 54 No Answer Given Incorrect

A 37-year-old woman with a history of depression was brought to hospital having been found in a state of
collapse. She had written a suicide note, and had an empty bottle of amitriptyline in her jacket.

On examination, her pulse was 135 beats/min and her BP was 105/50 mmHg.

Investigations:

serum potassium 4.5 mmol/L (3.5–4.9)


serum
18 mmol/L (20–28)
bicarbonate
serum urea 8.5 mmol/L (2.5–7.0)
serum creatinine 110 µmol/L (60–110)
   
sinus tachycardia; QRS duration 135 ms (80–120);
ECG
bursts of non-sustained ventricular tachycardia
What is the most appropriate initial intravenous treatment?

Answers

A: amiodarone
B: esmolol
C: magnesium
D: sodium bicarbonate
E: sodium chloride 0.9%

Correct answer: D
Explanation
Treatment of complications of tricyclic poisoning (hypotension and arrhythmias) consists of correction of
metabolic acidosis with sodium bicarbonate. Serum alkalinisation reduces binding of tricyclic antidepressants
to the myocardium.

Question 55 No Answer Given Incorrect

A 20-year-old man was found collapsed.

On examination, he had a GCS of 6. His pulse was 60 beats/min, his BP was 90/60 mmHg, and is respiratory
rate was 8 breaths/min. His muscle tone was reduced, his pupils were small but reacting and his plantar
responses were downgoing.

What is the most likely cause of this presentation?

Answers

A: diazepam
B: dihydrocodeine
C: diphenhydramine
D: MDMA ('ecstasy')
E: methanol

Correct answer: B
Explanation
The combination of reduced conscious level, reduced respiratory rate and small pupils (miosis) is a classic triad
for the opioid toxidrome.

Question 56 No Answer Given Incorrect

On removal of the renal arterial clamp following a donor kidney transplantation, the surgeon noted changes
suggestive of hyperacute rejection.

Which immunoglobulin is most likely to be responsible?

Answers

A: IgA
B: IgD
C: IgE
D: IgG
E: IgM
Correct answer: D
Explanation
Hyperacute rejection occurs because of pre-existing humoral immunity, which is IgG mediated.

Question 57 No Answer Given Incorrect

A 25-year-old woman presented with increasingly severe headache and two generalised seizures, 36 hours
after the normal vaginal delivery of her first baby at 40 weeks’ gestation. She had been treated with epidural
analgesia during labour.

On examination, her temperature was 37.6°C, she was drowsy, there was a mild left hemiparesis and both
plantar responses were extensor.

What is the most likely diagnosis?

Answers

A: bacterial meningitis
B: eclampsia
C: subarachnoid haemorrhage
D: venous sinus thrombosis
E: viral encephalitis

Correct answer: D
Explanation
The combination of headache, increasing drowsiness (suggesting increasing intracranial pressure), focal
neurological signs and seizures in a woman shortly post partum is highly suggestive of a venous sinus
thrombosis. None of the other options is impossible, but central venous sinus thrombosis is the first diagnosis
to consider and investigate.  

Question 58 No Answer Given Incorrect

The half-life of a novel anti-obesity drug exhibiting first-order kinetics was calculated to be 4 hours.

What percentage of the drug will be eliminated 20 hours after ingestion?

Answers

A: 75%
B: 80%
C: 90%
D: 97%
E: 100%
Correct answer: D
Explanation
After five half-lives, 97% of the drug will have been eliminated (50%+25%+12.5%+6.25%+3.125%)

Question 59 No Answer Given Incorrect

A 16-year-old boy with type 1 diabetes mellitus was treated with a biphasic insulin preparation but
achievement of good blood glucose control proved difficult. He was offered treatment with the insulin
analogue, insulin lispro.

Which characteristic of insulin lispro might improve his glycaemic control?

Answers

A: low incidence of hypoglycaemia


B: low incidence of lipoatrophy at the injection site
C: low risk of immunogenic reaction
D: rapid onset of action
E: small injection volume

Correct answer: D
Explanation
Insulin lispro is a rapid-acting human insulin analogue and leads to lower post-prandial glucose
concentrations than biphasic insulin.

Question 60 No Answer Given Incorrect

A 74-year-old woman presented with distressing restlessness in her legs, particularly at night. She usually had
to get out of bed several times to reduce her symptoms, and her sleep was being disturbed.

What is the most appropriate treatment?

Answers

A: clonazepam
B: co-beneldopa
C: propranolol
D: quinine
E: ropinirole

Correct answer: E
Explanation
The dopamine agonists ropinirole, pramipexole and rotigotine may be used for the treatment of moderate to
severe restless legs syndrome.

Question 61 No Answer Given Incorrect

A 32-year-old man was treated with combination chemotherapy for testicular cancer. 

Subsequent investigations confirmed a complete clinical remission.

What is the dominant cellular process that explains why this therapy was successful?

Answers

A: apoptosis
B: differentiation
C: mutagenesis
D: necrosis
E: senescence

Correct answer: A
Explanation
Apoptosis, programmed cell death, is the only mechanism that would precipitate cure without unacceptable
adverse effect. Differentiation, mutagenesis and senescence do not cause cell death, whereas necrosis causes
premature cell death by autolysis and has adverse clinical effects.

Question 62 No Answer Given Incorrect

In a large multicentre clinical trial to assess the effectiveness of a new antihypertensive drug, subjects
were randomised to be treated with either the new drug or an existing standard antihypertensive drug. The
main outcome criterion was the blood pressure after 2 months of treatment.

What is the most appropriate statistical technique to compare mean BP between the groups?

Answers

A: chi-squared test
B: Mann–Whitney U test
C: Pearson correlation coefficient
D: regression analysis
E: two-sample t-test

Correct answer: E
Explanation
The study is classed as “large” so a parametric test is acceptable, and both groups are likely to have the same
variance.

The two-sample t-test will compare the two sets of data better than Mann–Whitney U test.

Regression and correlation tests are not relevant. The chi-squared test is used for expected versus observed
results.

Question 63 No Answer Given Incorrect

A 71-year-old woman presented with a 2-hour history of severe pain and loss of vision in her right eye. She
had noticed halos around lights during the previous 24 hours.

On examination, there was ciliary vessel hyperaemia and a dilated unreactive pupil on the affected side.

What is the most likely diagnosis?

Answers

A: anterior uveitis
B: closed-angle glaucoma
C: diffuse scleritis
D: orbital cellulitis
E: retinal artery occlusion

Correct answer: B
Explanation
Acute closed-angle glaucoma is caused by a rapid or sudden increase in intraocular pressure. It can occur
with rapid dilatation of the pupil, causing failure of fluid drainage through the anterior chamber of the eye.
Symptoms include headaches, eye pain, nausea, halos and reduction of visual acuity. Ciliary vessel
hyperaemia is an early sign.

Question 64 No Answer Given Incorrect

A 16-year-old boy presented 30 minutes after sustaining a head injury. His parents reported that he had
fallen and hit the right side of his head on the ground. He had lost consciousness for a few seconds, then
recovered fully.

On examination, he was fully conscious and had no focal neurological signs. Two hours later, his conscious
level deteriorated.

What is the most likely diagnosis?


Answers

A: cerebral oedema
B: diffuse axonal injury
C: extradural haematoma
D: subarachnoid haemorrhage
E: subdural haematoma

Correct answer: C
Explanation
The mechanism of injury and the description of the ‘lucid period’ is highly suggestive of an extradural
haematoma. This type of injury is unlikely to lead to diffuse axonal injury and there would be no lucid period
with this. Subdural haematoma is more common in older people.

Question 65 No Answer Given Incorrect

A 55-year-old woman presented with a 2-day history of diplopia and progressive gait unsteadiness.

On examination, she had limited eye movements in all directions and her pupils were poorly reacting. Muscle
power in the limbs was normal but tendon reflexes were absent. There was prominent bilateral finger–nose
ataxia. Plantar responses were flexor. Sensation was normal.

What is the most likely diagnosis?

Answers

A: brainstem stroke
B: Miller Fisher syndrome
C: multiple sclerosis
D: myasthenia gravis
E: Wernicke encephalopathy

Correct answer: B
Explanation
Miller Fisher syndrome is the triad of ataxia, internal and external ophthalmoplegia and areflexia. It is often
associated with antibodies to GQ1b and is probably a variant of Guillain– Barré syndrome. The other answer
options do not have the same combination of signs.

Question 66 No Answer Given Incorrect

A 58-year-old man with congestive heart failure remained oedematous despite treatment with furosemide
120 mg daily.
Investigations:

serum sodium 134 mmol/L (137–144)


serum potassium 3.4 mmol/L (3.5–4.9)
serum urea 10.6 mmol/L (2.5–7.0)
serum creatinine 156 µmol/L (60–110)
What other finding is most likely?

Answers

A: high plasma aldosterone concentration


B: high serum cortisol concentration
C: low plasma angiotensin II concentration
D: low plasma atrial natriuretic peptide concentration
E: low plasma renin concentration

Correct answer: A
Explanation
In congestive cardiac failure, there is pathophysiological activation of the renin angiotensin aldosterone
system due to reduced renal perfusion pressure, which is identified within the nephron by the
juxtaglomerular apparatus. This leads to elevated levels of these hormones. Serum cortisol levels are not
usually significantly elevated in congestive cardiac failure. Atrial natriuretic peptide would be elevated in
congestive cardiac failure in response to increased atrial blood volume and atrial wall stretch. Option A is
therefore the correct answer.

Question 67 No Answer Given Incorrect

A 34-year-old woman, who was 14 weeks pregnant, had a 6-year history of Sjögren’s syndrome.

What autoantibody in this condition is most strongly associated with an increased risk of congenital heart
block in the fetus?

Answers

A: ANCA
B: anticardiolipin
C: anticentromere
D: anti-La
E: anti-Ro

Correct answer: E
Explanation
This question tests the knowledge of transplacental transfer of antibodies and consequences on the fetus.
Both anti-Ro and anti-La antibodies can be involved, but it is more common with anti-Ro antibodies. Both
antibodies are associated with neonatal lupus for the same reason.

Question 68 No Answer Given Incorrect

A 72-year-old man presented with a 2-day history of pain and swelling of the right knee.

Analysis of synovial fluid from the right knee confirmed the presence of calcium pyrophosphate crystals.

What is the microscopic appearance of these crystals?

Answers

A: needle-shaped with negative birefringence


B: needle-shaped with no birefringence
C: needle-shaped with positive birefringence
D: rhomboid with negative birefringence
E: rhomboid with positive birefringence

Correct answer: E
Explanation
Calcium pyrophosphate crystals cause pseudogout, and are seen on microscopy as positively birefringent
rhomboid crystals. Gout crystals are needle-shaped with negative birefringence.

Question 69 No Answer Given Incorrect

A 70-year-old man presented with a 2-week history of persistent fever. Six weeks previously, he had
undergone colonoscopy and biopsy for a suspected carcinoma of the colon, and was awaiting surgery. He
had a history of moderate mitral regurgitation caused by mitral valve prolapse.

Investigations:

echocardiogram a vegetation on the anterior leaflet of the mitral valve


What is the most likely causative organism?

Answers

A: Bartonella henselae
B: coagulase-negative streptococci
C: Staphylococcus aureus
D: Streptococcus gallolyticus (S. bovis)
E: Yersinia enterocolitica

Correct answer: D
Explanation
Streptococcus gallolyticus (S. bovis) is the best answer. S. gallolyticus endocarditis is classically associated with
colonic carcinoma and adenoma, and also with chronic liver disease.

Bartonella henselae causes cat scratch fever, and most patients with this uncommon form of endocarditis will
have had exposure to cats.

Staphylococcus aureus is an increasingly common cause of endocarditis, often seen in patients with early
endocarditis following heart valve surgery, as well as in patients with a history of intravenous drug use, or
with indwelling cannulae such as dialysis lines.

Coagulase-negative streptococcal endocarditis is the commonest cause of native valve endocarditis but is a
less likely cause than S. gallolyticus in this scenario.

Yersinia enterocolitica endocarditis is rare in comparison with the other options.

Question 70 No Answer Given Incorrect

A 75-year-old woman presented with a 2-month history of generalised malaise and headaches that had
disturbed her sleep. For the past 24 hours, she had been unable to see with her right eye. She had a history
of glaucoma and was using regular timolol eye drops.

On examination of the fundus, there was a swollen, pale right optic disc and a normal left optic disc.

What is the most likely diagnosis?

Answers

A: central retinal vein occlusion


B: closed-angle glaucoma
C: giant cell arteritis
D: optic neuritis
E: raised intracranial pressure

Correct answer: C
Explanation
The clinical picture suggests a right ischaemic optic neuropathy. The pain and malaise in someone of this age
would be highly suggestive of an arteritic process and giant cell arteritis should be suspected in all cases
presenting in this manner.

Question 71 No Answer Given Incorrect


A 66-year-old man, who was undergoing maintenance haemodialysis through a subcutaneous tunnelled
catheter, presented with fever, chills and rigors during haemodialysis.

On examination, his temperature was 38.6°C, his pulse was 105 beats/min and his BP was 100/60 mmHg.
Examination of his respiratory and cardiovascular systems was normal.

What is the most likely cause of his infection?

Answers

A: coagulase-negative streptococci
B: Escherichia coli
C: Pseudomonas aeruginosa
D: Staphylococcus epidermidis
E: Streptococcus pneumoniae

Correct answer: D
Explanation
Staphylococcus epidermidis or coagulase-negative staphylococcal infections are common infections with
vascular dialysis catheters. The other options are unusual.

Question 72 No Answer Given Incorrect

A 72-year-old woman with rheumatoid arthritis presented with dysuria, rigors and high fever of 2 days’
duration. Her regular medication comprised methotrexate, prednisolone and paracetamol.

On examination, her temperature was 39.0°C and she was tender in both flanks. Her BMI was 34.0 kg/m2
(18.0–25.0).

Investigations:

urine culture coliforms, sensitive to ciprofloxacin


Which condition is most likely to be precipitated by the use of ciprofloxacin?

Answers

A: gastric ulceration
B: glucose intolerance
C: hepatitis
D: pneumonitis
E: tendinopathy

Correct answer: E
Explanation
Tendon damage including rupture can occur rarely after use of quinolones. Quinolones are contraindicated in
patients with a history of tendon disorders related to quinolone use.

Question 73 No Answer Given Incorrect

A 35-year-old woman presented with a 6-month history of episodes of sweating, joint pain and headaches.

On examination, her BP was 160/90 mmHg. 

Investigations:

fasting plasma glucose 8.1 mmol/L (3.0–6.0)


A clinical diagnosis of acromegaly was suspected.

What additional investigation is most likely to confirm the diagnosis?

Answers

A: fasting growth hormone


B: growth hormone suppression test
C: insulin tolerance test
D: MR scan of pituitary
E: serum IGF-1

Correct answer: B
Explanation
Failure of growth hormone levels to be suppressed on a growth hormone suppression test indicates excess
production. Growth hormone is released in a pulsatile manner; therefore, an isolated sample is not
appropriate, IGF-1 levels are not diagnostic, insulin tolerance test assesses growth hormone deficiency and an
MR scan of pituitary will not reveal growth hormone excess.

Question 74 No Answer Given Incorrect

A 25-year-old man presented with a 3-month history of hearing two people commenting on his actions. For
the past 2 years, he had become increasingly withdrawn and had neglected his self-care. He had graduated
from university at 21 years of age, but had never been employed and had few friends. He had taken an
overdose of paracetamol 2 months previously. He had smoked cannabis regularly for 5 years. He drank 60
units of alcohol per week.

On examination, he made poor eye contact and his speech was disjointed.

What is the most likely diagnosis?


Answers

A: alcohol-related hallucinosis
B: borderline personality disorder
C: cannabis-induced psychosis
D: paranoid schizophrenia
E: psychotic depression

Correct answer: D
Explanation
Diagnosis of schizophrenia is supported by first-rank symptoms (voices discussing him), social withdrawal and
neglect (negative symptoms) and the patient’s age and time course. Disjointed speech reflects thought
disorder, also found in schizophrenia. Alcohol-related hallucinosis is rare and occurs after years of very heavy
alcohol use. The finding of graduating from university and then declining is against any form of personality
disorder, which would have been developing through childhood and adolescence. Psychotic depression is
unlikely because there is evidence for schizophrenia and, in psychotic depression, voices tend to be very
simple and elemental (noises or occasional words). While cannabis use is associated with schizophrenia, a
pure cannabis-related psychosis would tend to be more acute and he has been using cannabis for 5 years.

Question 75 No Answer Given Incorrect

A 56-year-old man presented with double vision

On examination, he had bilateral partial ptosis, diplopia and truncal muscle weakness.

Investigations:

chest X-ray widened mediastinum


 What is the most likely diagnosis?

Answers

A: dermatomyositis
B: Lambert–Eaton myasthenic syndrome
C: malignant meningitis
D: mononeuritis multiplex
E: myasthenia gravis

Correct answer: E
Explanation
The clinical presentation is entirely consistent with myasthenia gravis and the widened mediastinum would be
consistent with thymoma which is often associated with paraneoplastic myasthenia gravis
Question 76 No Answer Given Incorrect

A 63-year-old woman with a history of CREST syndrome developed progressive breathlessness.

On auscultation of the chest, she had bilateral basal crackles.

Investigations:

chest X-ray lower zone shadowing


What is the most likely diagnosis?

Answers

A: cryptogenic organising pneumonia


B: interstitial lung disease
C: left ventricular failure
D: progressive massive fibrosis
E: tuberculosis

Correct answer: B
Explanation
Patients with autoimmune disease develop interstitial lung disease, usually non-specific interstitial
pneumonitis (NSIP). The other conditions are not associated with CREST syndrome, although
immunosuppressed patients can develop cryptogenic organising pneumonia or get reactivation of
tuberculosis.

Question 77 No Answer Given Incorrect

A 75-year-old woman with hypertension had been treated with bendroflumethiazide for the past 5 years.

On examination, her BP was 125/78 mmHg.

Investigations:

serum sodium 140 mmol/L (137–144)


serum potassium 3.2 mmol/L (3.5–4.9)
serum creatinine 67 µmol/L (60–110)
What is the most likely mechanism for the hypokalaemia?

Answers

A: increased loss in the distal tubule


B: increased potassium secretion in the proximal tubule
C: opening of potassium channels in the principal cells
D: reduced aldosterone secretion
E: reduced potassium reabsorption in the loop of Henle

Correct answer: A
Explanation
Thiazide diuretics act on the distal tubule by blocking the thiazide-sensitive Na+-Cl− symporter. The increased
sodium delivery to the distal part of the distal tubule causes increased potassium loss, due to stimulation by
sodium of the aldosterone-sensitive sodium pump, thereby enhancing sodium reabsorption in exchange for
potassium (and hydrogen) ion.

Question 78 No Answer Given Incorrect

A 25-year-old injecting drug user presented with an injection-site abscess.

On examination, his temperature was 38.5°C and there was a pansystolic murmur.

What organism is most likely to be cultured from both the abscess and the blood?

Answers

A: Clostridium novyi
B: Klebsiella pneumoniae
C: Staphylococcus aureus
D: Staphylococcus epidermidis
E: Streptococcus anginosus (S. milleri group)

Correct answer: C
Explanation
A straightforward test of knowledge. The patient has an injection-site abscess and right-sided endocarditis.
Staphylococcus aureus is the most likely cause of this presentation.

Question 79 No Answer Given Incorrect

A 30-year-old doctor presented following a needlestick injury sustained while treating a patient with recently
diagnosed HIV infection. The patient was hepatitis C negative and had been vaccinated against hepatitis B
virus, and was not taking any antiretroviral treatment.

What is the most appropriate post-exposure prophylaxis for the doctor?

Answers
A: single-drug antiretroviral treatment for 1 month
B: single-drug antiretroviral treatment for 3 months
C: three-drug antiretroviral treatment for 1 month
D: three-drug antiretroviral treatment for 3 months
E: two-drug antiretroviral treatment for 1 month

Correct answer: C
Explanation
Post-exposure prophylaxis (PEP) of HIV requires a three-drug antiretroviral regimen in the same way that
treatment requires three drug regimens. All of the trials of PEP in needlestick injury have been done with 28
days of treatment and this is established practice.

Question 80 No Answer Given Incorrect

A 21-year-old woman, who was undergoing chemotherapy for non-Hodgkin lymphoma, presented 2 days
after being in contact with her nephew, for 2 hours, on the day that he developed a chickenpox rash.

Investigations:

varicella serology negative


What is the most appropriate management?

Answers

A: advise her to seek treatment if she develops a chickenpox rash


B: interrupt chemotherapy for 2 weeks
C: oral aciclovir 800 mg five times daily for 1 week
D: varicella immunisation
E: varicella zoster immunoglobulin

Correct answer: E
Explanation
The patient is immunocompromised and, with negative varicella serology, is at risk of varicella infection.
Varicella zoster immunoglobulin transfers passive immunity and reduces the risk of acquiring infection.

If she develops chickenpox, there is a high risk of adverse outcome; therefore, management cannot wait until
she develops chickenpox. Interrupting chemotherapy will make no difference to outcome. Chickenpox in an
immunocompromised host is always treated intravenously. Although oral aciclovir is sometimes
recommended when there is a supply problem with varicella zoster immunoglobulin, it cannot prevent
chickenpox. Varicella immunisation is contraindicated because it is a live vaccine at present.

Question 81 No Answer Given Incorrect


A 21-year-old woman presented with watery diarrhoea and cramping abdominal pain, 5 days after arriving in
Mexico.

On examination, her temperature was 37.8°C and there was mild abdominal tenderness. There was some
mucus in the stools, but no blood.

What is the most likely pathogen?

Answers

A: Aeromonas hydrophila
B: Entamoeba histolytica
C: Escherichia coli (enterotoxigenic)
D: Giardia intestinalis
E: rotavirus

Correct answer: C
Explanation
Bacteria are the most common cause of travellers’ diarrhoea. Overall, the most common pathogen is
enterotoxigenic Escherichia coli, followed by Campylobacter jejuni, Shigella spp., and Salmonella spp.
Therefore, the answer is E. coli (enterotoxigenic) because it is the most likely cause of the presentation.

The other options can all cause diarrhoeal illness, although rotavirus would be unlikely in adults and
Entamoeba histolytica would usually be bloody.

Question 82 No Answer Given Incorrect

A 36-year-old HIV-positive man presented with a 1-week history of generalised pruritus. He had had
unprotected anal sex 2 months previously while on holiday in Spain. He had previously been vaccinated
against hepatitis B virus, with an adequate antibody response.

On examination, his temperature was 37.8°C and he was jaundiced. Examination was otherwise normal.

Investigations:

CD4 count 550 × 106/L (430–1690)


   
serum total bilirubin 99 µmol/L (1–22)
serum aspartate aminotransferase 754 U/L (1–31)
serum alkaline phosphatase 173 U/L (45–105)
   
anti-hepatitis A IgM antibody negative
What is the most likely diagnosis?

Answers

A: acute hepatitis C
B: acute hepatitis D
C: CMV infection
D: syphilis
E: toxoplasmosis

Correct answer: A
Explanation
The patient has acute hepatitis. Hepatitis C is the most likely cause in a patient with HIV who has had
unprotected anal sex. Hepatitis C is sexually transmitted in these circumstances. Hepatitis D is only clinically
active when co-infected with hepatitis B.

CMV, syphilis and toxoplasmosis can all cause hepatitis but are less likely in this clinical context.

Question 83 No Answer Given Incorrect

Twenty-six passengers on a cruise ship returning from the Mediterranean to Southampton developed acute
gastrointestinal illness over a period of 24 hours. All those affected had experienced persistent vomiting and
watery diarrhoea.

What is the most likely causative agent?

Answers

A: Campylobacter jejuni
B: Escherichia coli O157:H7
C: norovirus
D: rotavirus
E: Salmonella enteritidis

Correct answer: C
Explanation
Norovirus is the most common cause of viral gastroenteritis. It is transmitted by faecally contaminated food
or water, by person-to-person contact, and via aerosolisation of vomited virus and subsequent contamination
of surfaces. Outbreaks in cruise ships are common.

Campylobacter jejuni, Escherichia coli O157:H7 and Salmonella enteritidis pass less easily from person to
person and are less likely to cause an outbreak on a cruise ship. Rotavirus is most common in children.
Question 84 No Answer Given Incorrect

A 52-year-old man presented after having been found unconscious in the street.

On examination, he had a GCS of 5, a pulse of 110 beats/min and a BP of 95/65 mmHg. His respiratory rate
was 35 breaths/min and there were no focal neurological signs.

Investigations:

arterial blood gases, breathing oxygen 40%:  


PO2 11.5 kPa
PCO2 3.9 kPa (4.7–6.0)
pH 7.24 (7.35–7.45)
H+ 58 nmol/L (35–45)
serum bicarbonate 12 mmol/L (21–29)
What acid–base disturbance is most likely to be present?

Answers

A: metabolic acidosis
B: mixed metabolic acidosis and metabolic alkalosis
C: mixed metabolic acidosis and respiratory acidosis
D: mixed respiratory alkalosis and metabolic acidosis
E: respiratory acidosis

Correct answer: A
Explanation
The elevated pH confirms this is an acidosis, likely metabolic due to the very low bicarbonate. It is not
respiratory acidosis as the PCO2 and bicarbonate are both low.

No history of chronic respiratory disease is given, so this is unlikely to be a significant contributor to the
presenting complaint. There is no evidence of alkalosis, either respiratory or metabolic. The most likely
explanation for these results is a metabolic acidosis.

Question 85 No Answer Given Incorrect

A 33-year-old man presented with a 6-month history of cough and breathlessness that tended to deterioriate
as the week progressed and improve when he went on holiday. He was a lifelong non-smoker and worked as
a paint sprayer in a car factory.

Investigations:
chest X-ray normal
What further investigation would be most useful in establishing a diagnosis?

Answers

A: cardiorespiratory exercise test


B: histamine challenge
C: IgE antibody to isocyanate
D: serial peak expiratory flow rate measurements
E: transfer factor for CO (TLCO)

Correct answer: D
Explanation
The presentation suggests occupational asthma. Serial peak flow measurements would be helpful in
determining that his symptoms are worse at work with corresponding drop in the peak flow. The other tests
will not necessarily show a link to his work.

Question 86 No Answer Given Incorrect

A 54-year-old right-handed woman following the sudden onset of difficulty in reading.

Investigations:

CT scan of head recent left parietal lobe infarct


What additional feature is most likely to be present?

Answers

A: acalculia
B: confabulation
C: cortical deafness
D: expressive dysphasia
E: homonymous hemianopia

Correct answer: A
Explanation
Dominant parietal lobe lesions can cause a variety of problems including receptive dysphasia, dyslexia,
inattention and sensory impairments, and also Gerstmann syndrome, which is characterised by finger
agnosia, acalculia, agraphia and left–right limb disorientation.

Question 87 No Answer Given Incorrect


A 37-year-old woman with breast cancer had a family history of breast and ovarian cancer. Molecular genetic
testing revealed a BRCA1 mutation.

What is the normal function of BRCA1?

Answers

A: angiogenesis
B: apoptosis
C: cell adhesion
D: promotion of mitosis
E: tumour suppression

Correct answer: E
Explanation
BRCA1 is a tumour suppressor gene, coding a DNA repair protein.

Question 88 No Answer Given Incorrect

A 32-year-old woman presented with a 2-day history of fever and myalgia and a 6-hour history of headache,
photophobia and neck stiffness.

Investigations:

white cell count 10.4 × 109/L (4.0–11.0)


random plasma glucose 5.2 mmol/L
   
CSF:  
opening pressure 180 mmH2O (120–250)
total protein 0.60 g/L (0.15–0.45)
glucose 3.5 mmol/L (3.3–4.4)
cell count 76/µL (≤5)
lymphocyte count 46/μL (≤3)
neutrophil count 30/μL (0)
What is the most likely causative organism?

Answers

A: enterovirus
B: Listeria monocytogenes
C: Mycobacterium tuberculosis
D: Neisseria meningitidis
E: Streptococcus pneumoniae

Correct answer: A
Explanation
The CSF values are most in keeping with viral meningitis and enterovirus is the most probable cause. It is
probably early viral meningitis given that neutrophils are seen in addition to lymphocytes. She has no risk
factors for listeria. The CSF in listeria shows high protein, low glucose and lymphocytosis. The CSF in
tuberculous meningitis shows very high protein, low glucose and lymphocytosis. The CSF in Neisseria
meningitides and Streptococcus pneumoniae shows high protein, low glucose (<50% plasma), and a very
high neutrophil count.

Question 89 No Answer Given Incorrect

A clinical trial compared medical versus surgical treatment for stable angina. A large number of patients
withdrew from the study.

What type of analysis should be used to estimate the likely benefits in similar groups of patients in future?

Answers

A: intention-to-treat
B: meta-analysis
C: on-treatment
D: post-hoc
E: sensitivity

Correct answer: A
Explanation
Intention-to-treat is a common method of analysis in randomised trials and considered good practice. The
outcomes of those who withdrew are included in the final analysis according to the arm of the trial to which
they were originally assigned.

Question 90 No Answer Given Incorrect

A 46-year-old man presented within 1 hour of ingesting 40 tablets of slow-release theophylline.

What is the most appropriate initial management?

Answers

A: activated charcoal
B: alkaline diuresis
C: gastric lavage
D: observation only
E: whole-bowel irrigation

Correct answer: A
Explanation
Activated charcoal given within 1 hour adsorbs theophylline and reduces its absorption. Multiple doses of
activated charcoal may also be given to enhance the elimination of theophylline.

Question 91 No Answer Given Incorrect

A 20-year-old woman presented 24 hours after taking an overdose of 80 tablets of levothyroxine


100 micrograms.

On examination, she was clinically euthyroid.

What is the most appropriate treatment?

Answers

A: beta-adrenoceptor blockade
B: forced alkaline diuresis
C: haemodialysis
D: no treatment required
E: stomach washout

Correct answer: D
Explanation
This patient presented late following a levothyroxine overdose and she remained asymptomatic and did not
develop any complications; therefore, no treatment was required.

Question 92 No Answer Given Incorrect

A 21-year-old African man was found unconscious at home.

On examination, he was drowsy but rousable. His pulse was 120 beats/min and his BP was 110/80 mmHg.
His peripheral oxygen saturation was 98% (94–98), breathing air.

Investigations:

haemoglobin 150 g/L (130–180)


   
arterial blood gases, breathing air:  
PO2  11.9 kPa (11.3–
12.6)
PCO2  4.7 kPa (4.7–6.0)
pH 7.42 (7.35–7.45)
+
H       38 nmol/L (35–45)
base excess   –1.0 mmol/L (±2.0)
   
oxygen saturation by four-wavelength
55% (94–98)
spectrophotometry
What is the most likely explanation for the above findings?

Answers

A: carbon monoxide poisoning


B: faulty oximeter probe
C: haemoglobinopathy
D: methaemoglobinaemia
E: skin pigmentation

Correct answer: A
Explanation
There is a wide discrepancy between the peripheral oxygen saturation (98%) and the oxygen saturation by
four-wavelength spectrophotometry (55%).

The former passes two wavelengths of light to the photodetector and measures only oxyhaemoglobin and
deoxyhaemoglobin. It cannot distinguish oxyhaemoglobin from carboxyhaemoglobin, which accumulates in
carbon monoxide (CO) poisoning. The four-wavelength spectrophotometer, on the other hand, is able to do
so.

In CO poisoning, CO avidly binds with haemoglobin to form carboxyhaemoglobin. As a result, the amount of
oxyhaemoglobin falls.

The four-wavelength spectrophotometer accurately detects this fall of oxyhaemoglobin to 55%, but the two-
wavelength peripheral pulse oximeter will give a false elevated reading – 98% being the sum of
oxyhaemoglobin and carboxyhaemoglobin, which it cannot measure separately.

Question 93 No Answer Given Incorrect

An 80-year-old woman presented with a 6-month history of progressive breathlessness.

On examination, her chest was normal, but there was a large goitre.

Which investigation is most useful to determine whether the goitre is the cause of her breathlessness?
Answers

A: bronchoscopy
B: CT scan of neck and chest
C: FVC
D: respiratory flow–volume loop
E: X-ray of thoracic inlet

Correct answer: D
Explanation
Pulmonary function tests are the most sensitive indicator of goitre being the cause of her breathlessness. The
flow–volume loop will show marked reduction of the peak inspiratory and expiratory flow, unlike most
pulmonary causes of breathlessness. Her FVC will not be affected by the presence of goitre. Bronchoscopy,
CT scan and X-ray of thoracic inlet may have other uses in the diagnostic pathway but will not be the best
way to confirm that it is the goitre causing breathlessness.

Question 94 No Answer Given Incorrect

A 59-year-old man was found to have a squamous cell lung carcinoma in his right upper lobe. He was a
smoker and reported breathlessness on modest exertion. He also complained of chronic back pain.

What is the most useful investigation in assessing his suitability for surgical resection?

Answers

A: bone scan
B: bronchoscopy
C: cardiopulmonary exercise testing
D: PET-CT scan
E: ventilation/perfusion isotope lung scan

Correct answer: D
Explanation
Non-small cell lung carcinoma confined to the right upper lobe and amenable to curative right upper
lobectomy is suggested. However, distant bony metastases need to be excluded in view of the back pain,
albeit chronic in duration.

PET-CT scan is the superior option to bone scan as it is able to stage the mediastinum as well with a high
degree of accuracy. Complex preoperative evaluation for lung resection – cardiopulmonary exercise testing
and ventilation/perfusion isotope lung scan – should only be done if staging tests confirm resectability.

Question 95 No Answer Given Incorrect


A 58-year-old man was seen with progressive breathlessness in the outpatient clinic. He had undergone
coronary artery bypass grafting 3 years previously and this had been complicated by a resternotomy for
tamponade. He recovered well but had started to feel breathless and fatigued over the past 12 months. His
history included type 2 diabetes mellitus and hypertension. He was a current smoker.

A diagnosis of constrictive pericarditis was made.

What is the most common physical finding seen in this group of patients?

Answers

A: ascites
B: finger clubbing
C: hepatomegaly
D: pericardial knock
E: pleural effusion

Correct answer: C
Explanation
Hepatomegaly is the best answer. Hepatomegaly is the earliest and most consistent feature of pericardial
constriction of the options here.

Ascites (a later phenomenon than hepatomegaly) and pericardial knock (recognised in around 50% of cases)
are also features of pericardial constriction, making this a true “best-of” question. Finger clubbing is not a
feature of constriction, however, and pleural effusion is uncommon.

Question 96 No Answer Given Incorrect

A 22-year-old woman presented with acute chest pains, 2 days after the uncomplicated delivery of her first
child, and was found to have an anterior ST-elevation myocardial infarction. She had no conventional risk
factors for coronary artery disease.

What is the most likely aetiology of her myocardial infarction?

Answers

A: atherosclerotic plaque rupture


B: coronary artery dissection
C: myocardial bridging
D: paradoxical embolism across patent foramen ovale
E: takotsubo cardiomyopathy

Correct answer: B
Explanation
Coronary artery dissection is the best answer, as it is a well-recognised cause of myocardial infarction in
relation to pregnancy. It is the most likely cause in this young patient who is in the early post-partum phase,
and who has no recognised risk factors for coronary atherosclerosis.

Atherosclerotic plaque rupture is the main alternative to coronary dissection here, and would still need to be
excluded, given that it is the most common cause of myocardial infarction otherwise. Myocardial bridging is a
rare cause of myocardial ischaemia or infarction, as it causes coronary constriction during ventricular systole,
whereas coronary blood flow is mainly diastolic. Paradoxical embolism across patent foramen ovale is a rare
though well-recognised cause of myocardial infarction, and may be more prevalent during pregnancy, which
is a hypercoagulable state. Takotsubo cardiomyopathy is also recognised during and after pregnancy, but is
more rare than atherosclerotic plaque rupture or coronary artery dissection.

Question 97 No Answer Given Incorrect

A 74-year-old man presented with severe rectal bleeding. He was normally fit and well and had had no
previous problems with his bowels.

On examination, he was pale. His pulse was 110 beats/min and his BP was 100/60 mmHg. Abdominal
examination was normal. Rectal examination showed fresh blood on the glove.

Investigations:

haemoglobin 85 g/L (130–180)


white cell count 7.5 × 109/L (4.0–11.0)
What is the most likely diagnosis?

Answers

A: bleeding diverticulum
B: carcinoma of colon
C: haemorrhoids
D: inflammatory bowel disease
E: Meckel's diverticulum

Correct answer: A
Explanation
The most common cause of such heavy bleeding in a 74-year-old is diverticular-disease related. It is very
unusual for cancer to bleed so heavily and there often is a preceding history of bleeding. Likewise,
haemorrhoidal bleeding rarely causes haemodynamic instability. Inflammatory bowel disease very unusually
leads to such heavy bleeding and there will often be a preceding history of symptoms.

Question 98 No Answer Given Incorrect


A 56-year-old woman with hypertension was found to have abnormal liver function tests when she attended
for routine review. She was taking bendroflumethiazide, ramipril and simvastatin.

On examination, her BMI was 30.0 kg/m2 (18.0–25.0).

Investigations:

serum total bilirubin 15 µmol/L (1–22)


serum alanine aminotransferase 65 U/L (5–35)
serum aspartate aminotransferase 75 U/L (1–31)
serum alkaline phosphatase 305 U/L (45–105)
serum gamma glutamyl transferase 150 U/L (4–35)
What investigation is most likely to establish the liver diagnosis?

Answers

A: antimitochondrial antibodies
B: CT scan of abdomen
C: MRCP
D: serum ferritin
E: ultrasound scan of abdomen

Correct answer: A
Explanation
Antimitochondrial antibodies are present in over 90% of patients with primary biliary cirrhosis (PBC). The
pattern of deranged liver function tests is cholestatic and likely indicates PBC. CT scan and MRCP are both
unhelpful in diagnosing PBC as it mainly affects small bile ducts. Serum ferritin will be unhelpful as she is
unlikely to have haemochromatosis. Ultrasound scan is indicated and helpful to exclude other associated
abnormalities such as fatty liver. However, it is non-specific and will not establish a diagnosis.

Question 99 No Answer Given Incorrect

A 72-year-old man presented with a 3-month history of worsening neck and right arm pain radiating down
the inner forearm, associated with numbness and tingling involving the index and middle fingers.

Examination of the right upper limb revealed reduced forearm pronation and wrist flexion, decreased
sensation in the index and middle fingers, and diminished triceps tendon reflex.

What is the most likely site of the lesion?

Answers

A: axillary nerve
B: C6 root
C: C7 root
D: median nerve
E: radial nerve

Correct answer: C
Explanation
The sensory and motor abnormalities described are as a result of a lesion at the C7 nerve root causing
sensory loss in the C7 and 8 dermatome and motor abnormalities attributed to the C7 myotome.

Question 100 No Answer Given Incorrect

A 78-year-old woman presented with a 9-month history of worsening urinary frequency and urge
incontinence.

Examination was normal.

Investigations:

post-void bladder scan 10 mL residual volume


Antagonism of which neurotransmitter is most likely to improve her symptoms?

Answers

A: acetylcholine
B: adrenaline
C: dopamine
D: noradrenaline
E: serotonin

Correct answer: A
Explanation
Contraction of the bladder detrusor muscle is stimulated by acetylcholine. Anticholinergic agents inhibit the
binding of acetylcholine to the cholinergic receptor, thereby suppressing involuntary bladder contraction of
any aetiology.

Question 101 No Answer Given Incorrect

A 78-year-old woman presented with chronic induration of the skin of both legs. She had a history of chronic
venous insufficiency and had previously been treated for venous ulceration.

On examination, she had bilateral reddish-brown skin pigmentation.


She was found to have lipodermatosclerosis.

What structure is implicated in the pathophysiology of this condition?

Answers

A: dermis
B: hypodermis
C: stratum basale
D: stratum corneum
E: stratum spinosum

Correct answer: B
Explanation
Lipodermatosclerosis or sclerosing panniculitis is an extensive induration of the skin, often in association with
leg ulcers and most commonly related to venous hypertension; it is a chronic inflammation, with fibrous
tissue that involves and replaces the hypodermis and, if extensive, the papillary dermis.

Question 102 No Answer Given Incorrect

An 89-year-old woman presented with general deterioration and worsening mobility. She had vascular
dementia, and a history of recent falls and urinary incontinence. She consumed 21 units of alcohol per week.

Her Waterlow score was 21/50.

Investigations:

serum albumin 30 g/L (37–49)


Which parameter is most likely to have contributed to the patient’s high Waterlow score?

Answers

A: alcohol consumption
B: cognitive status
C: continence
D: history of recent falls
E: serum albumin level

Correct answer: C
Explanation
The Waterlow score is a tool used to assess the risk of development of a pressure sore. It uses the following
parameters to risk-stratify the patient: body weight, nutritional status, continence, skin type, mobility, age
and sex. It may be used both in an acute setting or in care homes.
Question 103 No Answer Given Incorrect

A 68-year-old man became acutely breathless following admission with a 2-week history of nausea, vomiting
and diarrhoea. Despite fluid resuscitation, he had remained anuric 6 hours after admission.

On examination, his BP was 104/63 mmHg, his JVP was 10 cm above the sternal angle and his respiratory
rate was 32 breaths/min. He had bibasal respiratory crackles and pitting oedema in both ankles.

Investigations:

serum sodium 138 mmol/L (137–144)


serum potassium 5.6 mmol/L (3.5–4.9)
serum bicarbonate 12 mmol/L (20–28)
serum urea 29.0 mmol/L (2.5–7.0)
serum creatinine 640 µmol/L (60–110)
What is the most appropriate next step in management?

Answers

A: dopamine at renal dose


B: furosemide
C: haemodialysis
D: insulin and glucose 5%
E: sodium bicarbonate

Correct answer: C
Explanation
Haemodialysis is the right answer as he is already fluid overloaded. Furosemide will not work in this situation
to cause a diuresis as the patient is anuric, and the other options are not feasible or have no therapeutic
value given his pulmonary oedema.

Question 104 No Answer Given Incorrect

A 52-year-old woman presented with an episode of weakness of the left side of her face and left arm that
resolved after 3 hours. She was otherwise well and had no significant past medical history.

On examination, her BP was 110/75 mmHg.

Investigations:

ECG atrial fibrillation with a rate of 110 beats/min


   
echocardiogram structurally normal heart
Treatment was started with atenolol and warfarin.

She underwent a successful DC cardioversion 4 weeks later.

How long should she continue with warfarin therapy?

Answers

A: 4 weeks
B: 6 months
C: 1 year
D: 3 years
E: lifelong

Correct answer: E
Explanation
This woman has a history of transient ischaemic attack, which puts her at high risk of recurrent embolic
episodes even after a successful cardioversion. She would score 3 on the CHA2DS2-VASc score and
anticoagulation should be considered lifelong following a discussion about stroke and bleeding risk.

Question 105 No Answer Given Incorrect

A 72-year-old man presented following a haematemesis. He had a history of alcohol-related liver cirrhosis.

On examination, his pulse was 110 beats/min and his BP was 90/60 mmHg.

Investigations:

haemoglobin 104 g/L (130–180)


What is the most appropriate first step in management?

Answers

A: blood transfusion
B: intravenous albumin
C: intravenous gelatin
D: intravenous omeprazole
E: intravenous sodium chloride 0.9%

Correct answer: E
Explanation
Resuscitation with a crystalloid, such as sodium chloride 0.9% or compound sodium lactate intravenous
solution, is the best management in shocked patients with or without liver disease. Use of albumin is not
recommended in acute shock due to gastrointestinal bleeding, nor is there any role for gelatins any longer.
Blood transfusion may be required at some stage, if there is a further drop in haemoglobin but not before
fluid resuscitation. There is no evidence for an intravenous proton pump inhibitor before endoscopy.

Question 106 No Answer Given Incorrect

A 57-year-old woman attended the diabetes-nephrology clinic for annual review. Her history included
hypertension, type 2 diabetes mellitus and diabetic retinopathy.

On examination, her BP was 168/98 mmHg. Urinalysis showed protein 1+, glucose 1+.

What is the most useful investigation to determine her future cardiovascular risk?

Answers

A: 24-h urine collection for protein estimation


B: urinary lipid peroxide test
C: urine albumin:creatinine ratio
D: urine microscopy
E: urine protein:creatinine ratio

Correct answer: C
Explanation
Proteinuria is an independent predictor of cardiovascular risk in patients with diabetes mellitus and/or
hypertension.

Screening for proteinuria by performing a spot urine albumin:creatinine ratio is the most commonly
performed investigation to assess this risk. From a pathophysiological perspective, albuminuria reflects
glomerular protein leak whereas proteinuria more broadly reflects protein excretion into the urine from
anywhere along the nephron. Additionally, a urine albumin:creatinine ratio is more sensitive than a urine
protein:creatinine ratio for detecting low levels of proteinuria (e.g. negative or protein 1+ on reagent strip
urinalysis) and is the NICE recommended method for screening for diabetic nephropathy. This makes urine
albumin:creatinine ratio a better option than protein:creatinine ratio.

A 24-h urine collection is more cumbersome for patients to perform and, as they are commonly incompletely
performed are often not accurate. Urinary lipid peroxide is a potential biomarker to assess oxidative stress. It
is not routinely used in clinical practice and is obviously incorrect. Urine microscopy will not give information
on proteinuria.

Question 107 No Answer Given Incorrect

A 77-year-old woman presented with a 1-day history of fever and tiredness. She had been discharged after
surgery for a fractured neck of femur 1 week previously. She had received a 2-unit red cell transfusion before
discharge. She was taking nitrofurantoin for a urinary tract infection and rivaroxaban as thromboprophylaxis.

Investigations (before transfusion):

haemoglobin 78 g/L  (115–165)


Investigations (current):

haemoglobin 76 g/L  (115–165)


MCV 101 fL (80–96)
white cell count 10.8 × 109/L (4.0–11.0)
platelet count 207 × 109/L (150–400)
reticulocyte count 146 × 109/L (25–85)
   
direct antiglobulin test strongly positive
What is the most likely cause of her anaemia?

Answers

A: autoimmune haemolytic anaemia


B: bleeding
C: delayed haemolytic transfusion reaction
D: drug-induced haemolytic anaemia
E: urinary tract infection

Correct answer: C
Explanation
The history given in the stem makes clear that the current blood sample was taken 1 week after the
transfusion. The investigations show that there has been an inadequate rise in haemoglobin with a mild
macrocytosis, elevated reticulocyte count (which causes the increased MCV) and a positive DAT (telling you
that there is antibody coating the red cells). This would all be consisted with a delayed haemolytic transfusion
reaction. Autoimmune haemolytic anaemia would be consistent with the results seen, though would be less
likely in this context as it would indicate the development of a second pathology unrelated to the information
in the stem. Blood loss could cause a fall in the haemoglobin, though would not be associated with a positive
DAT. A drug-induced haemolytic anaemia related to nitrofurantoin is feasible, though the haemolysis would
likely be more significant than the relatively mild picture noted here. Urinary tract infection is not appropriate
as infection-mediated haemolysis would be less likely than delayed haemolytic transfusion reaction in this
stem.

Question 108 No Answer Given Incorrect

A 75-year-old man was admitted with a 10-day history of generalised reddening of his skin, accompanied by
shivering and thirst. He had a history of psoriasis.
On examination, his pulse was 110 beats/min and his BP was 110/60 mmHg. He had dependent oedema. His
skin was uniformly erythematous and scaly.

What is the most appropriate initial treatment?

Answers

A: oral furosemide
B: oral prednisolone
C: topical coal tar solution
D: topical dithranol ointment
E: topical white soft paraffin

Correct answer: E
Explanation
Based on the fact that this patient’s skin is uniformly erythematous and scaly, and he has a history of
psoriasis, the most likely diagnosis is erythrodermic psoriasis. This inflammation in the skin can sometimes
result in oedema of the legs. Initial treatment is with topical white soft paraffin smeared all over the skin,
which helps to make the skin more comfortable, partially reduces the inflammation, improves the skin barrier
(which is generally defective in erythroderma), thus helping to prevent entry of bacteria through the skin (and
prevent septicemia), and helps to reduce water loss from the skin (and prevent dehydration). While using
topical white soft paraffin, relevant investigations can be performed to determine the cause of the
erythroderma and to determine which systemic therapy can be employed to treat the cause of the
erythroderma.

The other options are incorrect because topical coal tar and dithranol irritate very inflamed skin and can
exacerbate the inflammation, so are best avoided; oral prednisolone is contraindicated because of the risk of
precipitating generalised pustular psoriasis, and oral furosemide is unhelpful (because the oedema is due to
the intense skin inflammation, which is not treatable by furosemide) and may cause further dehydration.

The vapour from soft paraffin is flammable. Patients, especially smokers, should be advised of this and told
never to allow a naked flame next to their skin while using it.

Question 109 No Answer Given Incorrect

A 24-year-old woman presented with shortness of breath of sudden onset. She had been at a nightclub and
had been drinking wine but denied taking any illicit substances. She had no relevant medical history. She was
a non-smoker.

On examination, her temperature was 37.0°C, her pulse was 100 beats/min and her BP was 110/70 mmHg.
Her respiratory rate was 32 breaths/min and her oxygen saturation was 98% (94–98) breathing air.

Investigations:
arterial blood gases, breathing air:  
PO2 12.9 kPa (11.3–12.6)
PCO2 3.8 kPa (4.7–6.0)
pH 7.44 (7.35–7.45)
H+ 36 nmol/L (35–45)
bicarbonate 19 mmol/L (21–29)
base excess –2.0 mmol/L (±2.0)
   
chest X-ray normal
What is the most likely diagnosis?

Answers

A: asthma
B: hyperthyroidism
C: hyperventilation
D: pulmonary embolism
E: vocal cord dysfunction

Correct answer: C
Explanation
Mild hyperoxaemia (while breathing room air) and respiratory alkalosis associated with sudden onset of
dyspnoea and tachypnoea is strongly suggestive of hyperventilation. Her supranormal PO2 argues against
asthma or pulmonary embolism.

Question 110 No Answer Given Incorrect

A 30-year-old man with Marfan syndrome presented with the sudden onset of interscapular pain.

Investigations:

CT scan of chest aortic dissection


Which protein is most commonly abnormal in this syndrome?

Answers

A: collagen
B: desmin
C: elastin
D: fibrillin
E: myosin

Correct answer: D
Explanation
Collagen: abnormalities in collagen metabolism are associated with vascular abnormalities such as
spontaneous coronary dissection.

Desmin: desminopathy, a disease caused by dysfunctional mutations in desmin, presents as cardiomyopathy


and may cause arrhythmias.

Elastin: aortic supravalvular stenosis is the most common cardiac anomaly, present in 64% of patients with
Williams–Beuren Syndrome (elastin gene locus microdeletion).

Fibrillin: abnormalities of fibrillin metabolism is seen in most patients and can present with aortic dissection.

Myosin: mutations in cardiac myosin-binding protein C (MyBPC) gene are associated with hypertrophic
cardiomyopathy (HCM).

Question 111 No Answer Given Incorrect

A 70-year-old man presented with acute abdominal pain.

At laparotomy, his bowel was found to be ischaemic from the lower part of the duodenum through to
midway along the transverse colon.

What artery is most likely to be occluded?

Answers

A: coeliac
B: ileocolic
C: inferior mesenteric
D: middle colic
E: superior mesenteric

Correct answer: E
Explanation
The superior mesenteric arises from the abdominal aorta and supplies the small intestine from the distal
duodenum as well as the large intestine as far as 2/3 of the transverse colon.

Question 112 No Answer Given Incorrect

A 21-year-old man presented to the emergency department after he had vomited a small amount of blood
earlier that day, following an episode of severe retching. He was otherwise well and was not taking any
medication.
On examination, his pulse was 95 beats/min and his BP was 115/85 mmHg.

Investigations:

haemoglobin 133 g/L (130–180)


prothrombin time 12.0 s (11.5–15.5)
   
serum urea 5.2 mmol/L (2.5–7.0)
serum gamma glutamyl transferase 25 U/L (<50)
What is the most appropriate next step in management?

Answers

A: admit for 24-h observation


B: admit for urgent upper gastrointestinal endoscopy
C: discharge with reassurance
D: refer for outpatient upper gastrointestinal endoscopy
E: refer for outpatient Helicobacter pylori breath test

Correct answer: C
Explanation
He is likely to have sustained a Mallory–Weiss tear based on the presentation of bleeding following retching.
This is a low-risk upper gastrointestinal (GI) bleed. His haemoglobin and urea are normal and he is
haemodynamically stable. He has no comorbidities. Using risk assessment tools for upper GI bleeding (such
as the Blatchford score), this patient would be very low risk and can be discharged home with advice to
present again if further bleeding occurs. Although some may refer the patient for endoscopy, this
investigation is not needed if there is a clear history suggestive of Mallory–Weiss tear. Helicobacter pylori
testing is not indicated as he has no symptoms suggestive of infection.
 

Question 113 No Answer Given Incorrect

A 21-year-old woman presented with a 12-month history of amenorrhoea following discontinuation of the
combined oral contraceptive pill, which she had taken for 2 years. She had undergone a normal menarche
and had had a regular menstrual cycle until she started training to run marathons.

On examination, her BMI was 18.0 kg/m2 (18.0–25.0). She had mild facial acne and normal secondary sexual
characteristics.

What is the most likely cause of her menstrual disturbance?

Answers
A: hypothalamic amenorrhoea
B: macroprolactinoma
C: polycystic ovary syndrome
D: post-contraceptive pill amenorrhoea
E: premature ovarian failure

Correct answer: A
Explanation
The question shows that the patient’s BMI is on the lower limit of the normal range and that she has been
training for marathons, implying weight loss with intense exercise which is known to cause the hypothalamus
to reduce production of gonadotropin-releasing hormone (GnRH). The clinical picture given does not fit
polycystic ovary syndrome, macroprolactinoma, premature ovarian failure or post-contraceptive pill
amenorrhoea.

Question 114 No Answer Given Incorrect

An 82-year-old man developed agitation and confusion, with visual hallucinations, 1 day after surgery for a
hip fracture. His family reported that he had been forgetful for the past year but was otherwise independent.

On examination, he was easily distracted and could not comply with the mental test examination.

What is the most likely diagnosis?

Answers

A: Alzheimer’s disease
B: delirium
C: dementia with Lewy bodies
D: mania
E: schizophrenia

Correct answer: B
Explanation
The context and symptoms suggest delirium as the cause. Delirium is characterised by acute onset of
confusion and fluctuating course, inattention, disorganised thinking and altered level of consciousness. The
history is too short to suggest that this is dementia. There is also nothing in the history to suggest a
psychiatric condition.
 

Question 115 No Answer Given Incorrect

A 30-year-old man presented with a 2-year history of an intermittent facial rash.


On examination, there was an erythematous, scaly rash involving the nasolabial folds and eyebrows, and the
skin overlying the sternum. There was mild scaling of the scalp. The nails were normal.

What is the most likely diagnosis?

Answers

A: asteatotic eczema
B: atopic eczema
C: contact dermatitis
D: psoriasis
E: seborrhoeic dermatitis

Correct answer: E
Explanation
The description of an erythematous scaly rash involving the nasolabial folds, eyebrows and mid sternum, with
mild scaling of the scalp is typical of seborrhoeic dermatitis, which is why this is the most likely diagnosis in
this list of answers.

Question 116 No Answer Given Incorrect

A 30-year-old man with type 1 diabetes mellitus presented with onychomycosis affecting four digits of the
left foot. This had been unresponsive to a 6-month course of topical amorolfine.

Investigations:

nail culture Trichophyton rubrum


What is the most appropriate next treatment?

Answers

A: oral fluconazole
B: oral nystatin
C: oral terbinafine
D: topical amphotericin
E: topical nystatin

Correct answer: C
Explanation
This is in keeping with NICE guidelines for the management of athlete’s foot. In an adult with severe or
extensive disease, or when topical treatment has failed, options include an oral antifungal treatment. If an
oral antifungal is indicated, prescribe terbinafine, griseofulvin, or itraconazole, based on consideration of
their contraindications, cautions, and drug interactions. Oral fluconazole and oral ketoconazole are not
recommended. Terbinafine is generally better tolerated and has fewer interactions than griseofulvin and
itraconazole, but it is not suitable for all people (such as children and people with chronic or active liver
disease).

Question 117 No Answer Given Incorrect

A 33-year-old man presented with dyspepsia and, on serological testing, was found to have Helicobacter
pylori infection.

With which condition is H. pylori infection most strongly associated?

Answers

A: duodenal ulcer
B: gastric carcinoma
C: gastric ulcer
D: gastrooesophageal reflux
E: non-ulcer dyspepsia

Correct answer: A
Explanation
Duodenal ulcers have a strong association with Helicobacter pylori, with over 90% of patients with duodenal
ulcers being H. pylori positive. There is also an association with gastric ulceration and carcinoma but to a
lesser degree. Gastrooesophageal reflux is not particularly associated with H. pylori since infection tends to
reduce gastric acid secretion. Non-ulcer dyspepsia can occur with H. pylori but the association is more
complex and the conditions do not always co-exist.

Question 118 No Answer Given Incorrect

A 43-year-old man, who had recently emigrated from Bangladesh, presented with a 3-month history of
breathlessness and productive cough that had markedly worsened over the past week.

On examination, his pulse was 105 beats/min and his BP was 95/54 mmHg. His JVP was 6 cm above the
sternal angle. Coarse crackles were heard in the right upper zone.

Investigations:

5-cm pericardial effusion with diastolic collapse of right


echocardiogram
atrium and right ventricle
What is the most likely abnormality to be found in his pulse?

Answers

A: anacrotic pulse
B: collapsing pulse
C: pulsus alternans
D: pulsus bisferiens
E: pulsus paradoxus

Correct answer: E
Explanation
Anacrotic pulse: a low-volume pulse seen in patients with aortic stenosis.

Collapsing pulse: a finding in patients with moderate-to-severe aortic regurgitation.

Pulsus alternans: usually found in severe left ventricular dysfunction as a result of cardiomyopathy, coronary
artery disease, systemic hypertension, and aortic stenosis.

Pulsus bisferiens: a finding in patients with moderate-to-severe aortic regurgitation; also occurs in patients
with combined aortic stenosis and regurgitation.

Pulsus paradoxus: this is the only sign that would fit with the clinical scenario of a patient who is likely to
have tamponade.

Question 119 No Answer Given Incorrect

A 75-year-old woman presented following an impulsive overdose of three times her normal daily dose of
warfarin. Her husband had died 4 years previously. She did not normally drink much alcohol, but had
consumed 6 units on the day of the overdose.

What is most predictive of completed suicide in the next week?

Answers

A: age over 70 years


B: alcohol consumption
C: amount of warfarin taken
D: female sex
E: loss of husband

Correct answer: A
Explanation
The risk factors for completed suicide are male rather than female, older age is a strong risk factor, alcohol is
associated with both attempted and completed suicide but not as great a risk as this patient’s age. The
amount of tablets is a poor indicator as it depends on what the patient believed would be the outcome of
the overdose. Bereavement alone is not a strong risk factor.
Question 120 No Answer Given Incorrect

A 55-year-old man presented after a recent myocardial infarction. He was concerned about the risk to other
family members after reading a newspaper article about association studies that mentioned ischaemic heart
disease and single nucleotide polymorphism (SNP) genotyping.

What is the most appropriate definition of an SNP?

Answers

A: alteration in nucleotide sequence that causes a change in amino acid sequence


B: alteration in nucleotide sequence that does not cause a change in amino acid sequence
C: deletion of a single nucleotide
D: repetition of a nucleotide two or more times
E: substitution of a nucleotide for any other nucleotide

Correct answer: E
Explanation
Single nucleotide polymorphisms (SNPs) are the most common type of genetic variation. They are most
commonly found in non-coding DNA between genes. SNPs in regulatory or coding regions of DNA are more
clinically significant.

Question 121 No Answer Given Incorrect

An 83-year-old man presented with a 6-month history of pain in both legs on walking, which was relieved by
a short period of rest, particularly sitting down. He had a past history of myocardial infarction.

On examination, his foot pulses were present. There was crepitus in both knees and both ankle tendon
reflexes were absent.

What is the most likely cause of his symptoms?

Answers

A: osteoarthritis of the knees


B: peripheral arterial disease
C: peripheral polyneuropathy
D: spinal stenosis
E: venous insufficiency

Correct answer: D
Explanation
This question is testing the knowledge of the candidate regarding claudication pain in the legs. The
commonest cause is vascular claudication. The confounder here is the history of myocardial infarction,
indicating a high risk for macrovascular disease. Clinical examination confirms that feet pulses rule out a
vascular cause of claudication. The correct answer is spinal stenosis, which can mimic vascular claudication.
Nerve pressure is dynamic and hence symptoms resolve with rest. For the same reason, neurological
examination can be normal. Osteoarthritis, peripheral neuropathy and venous insufficiency do not cause
claudication.

Question 122 No Answer Given Incorrect

A 24-year-old man presented with a 2-week history of leg swelling. He had experienced an upper respiratory
tract infection 1 month previously, but had not required antibiotic therapy. He had no significant medical
history. A maternal uncle had undergone a successful kidney transplant.

On examination, there was peripheral oedema to the thighs bilaterally. Urinalysis showed blood 1+, protein
4+.

Investigations:

serum urea 6.6 mmol/L (2.5–7.0)


serum creatinine 102 µmol/L (60–110)
eGFR (MDRD) >60 mL/min/1.73 m2 (>60)
serum albumin 22 g/L (37–49)
What is the most likely diagnosis?

Answers

A: Alport syndrome
B: autosomal dominant polycystic kidney disease
C: IgA nephropathy
D: membranoproliferative glomerulonephritis
E: minimal change nephropathy

Correct answer: E
Explanation
This patient has nephrotic syndrome (characterised by the triad of oedema, proteinuria (>3g/24h) and
hypoalbuminaemia). Of the options listed minimal change disease is the most likely cause of nephrotic
syndrome in this patient. A trigger, such as a viral respiratory tract infection, can occur before the acute
presentation.
IgA nephropathy and membranoproliferative glomerulonephritis can present with nephrotic syndrome,
although these diseases much more commonly present with progressive renal impairment and haematuria
with non-nephrotic proteinuria. Minimal change disease is a much more common cause of nephrotic
syndrome than these disorders.

Autosomal dominant polycystic kidney disease and Alport syndrome are respectively, the first and second,
most common hereditary kidney diseases. They are not causes of nephrotic syndrome and so are not correct
options.

Question 123 No Answer Given Incorrect

A 42-year-old woman was referred following a health check. She was asymptomatic but had been found to
have hyponatraemia. She had a history of gastrooesophageal reflux treated with omeprazole.

On examination, she had no postural drop in BP.

Investigations:

serum sodium 131 mmol/L (137–144)


serum potassium 4.5 mmol/L (3.5–4.9)
serum creatinine 88 µmol/L (60–110)
random plasma glucose 4.6 mmol/L
serum osmolality 271 mosmol/kg (278–300)
   
urine osmolality 735 mosmol/kg (100–1000)
urinary sodium 36 mmol/L
   
serum cortisol (09.00 h) 524 nmol/L (320–700)
What additional information would be most useful to support a diagnosis of syndrome of inappropriate
antidiuretic hormone?

Answers

A: daily fluid intake


B: daily urine output
C: no further information needed
D: serum cholesterol levels
E: thyroid function tests

Correct answer: E
Explanation
Diagnosis of syndrome of inappropriate antidiuretic hormone (SIADH) can only be made in the presence of
normal thyroid, adrenal and kidney function.

Question 124 No Answer Given Incorrect

A 19-year-old man presented with a 6-month history of increasing social withdrawal, having become
convinced that he could hear his innermost thoughts being spoken by people whose voices he did not
recognise. He spent hours pacing about in his room. Substance misuse was excluded and a diagnosis of
schizophrenia was made. He was treated with risperidone.

What receptors are chiefly involved in mediating the antipsychotic effects of risperidone?

Answers

A: alpha-adrenoceptor (α1)
B: dopamine (D2)
C: histamine (H1)
D: muscarinic (M1)
E: serotonin (5-HT2)

Correct answer: B
Explanation
The primary mechanism of action of antipsychotic drugs is dopamine receptor antagonism.

Question 125 No Answer Given Incorrect

A 68-year-old woman with atrial fibrillation presented for DC cardioversion. The procedure resulted in
successful restoration of sinus rhythm.

Which drug would be most likely to maintain sinus rhythm following this procedure?

Answers

A: amiodarone
B: digoxin
C: diltiazem
D: sotalol
E: verapamil

Correct answer: A
Explanation
Class IA (disopyramide phosphate, quinidine sulfate), Class IC (flecainide acetate, propafenone hydrochloride)
and Class III (amiodarone, dofetilide, dronedarone, sotalol hydrochloride) drugs reduce recurrence of atrial
fibrillation. Amiodarone and propafenone are superior to sotalol in maintaining long-term normal sinus
rhythm in patients with atrial fibrillation. Amiodarone is superior to propafenone. All but amiodarone and
propafenone increases pro-arrhythmia.

Question 126 No Answer Given Incorrect

A 79-year-old man with renal cell carcinoma with metastases involving his lungs and liver was treated with
morphine sulfate for pain control and clarithromycin for a respiratory infection.

Investigations:

serum urea 18.0 mmol/L (2.5–7.0)


serum creatinine 200 µmol/L (60–110)
serum albumin 28 g/L (37–49)
serum total bilirubin 20 µmol/L (1–22)
serum alanine aminotransferase 300 U/L (5–35)
serum alkaline phosphatase 1050 U/L (45–105)
What is most likely to cause morphine toxicity in this patient?

Answers

A: cholestasis
B: decreased plasma protein binding
C: hepatocellular dysfunction
D: renal impairment
E: treatment with clarithromycin

Correct answer: D
Explanation
Morphine is metabolised to active glucuronide metabolites, which are excreted renally. Renal impairment
leads to accumulation of morphine-6-glucuronide, which is a potent opioid agonist with a higher affinity
than morphine.

Question 127 No Answer Given Incorrect

A 32-year-old man presented with weight loss and feeling generally unwell. On direct questioning, he had
been having dizzy spells, especially on sudden changes in posture.

On examination, he was tanned. There was a postural drop in his BP.


What is the most likely biochemical finding?

Answers

A: 24-h urinary free cortisol 60 nmol (55–250)


B: fasting plasma glucose 3.2 mmol/L (3.0–6.0)
C: plasma adrenocorticotropic hormone 95 pmol/L (3.3–15.4)
D: serum potassium 4.6 mmol/L (3.5–4.9)
E: serum sodium 148 mmol/L (137–144)

Correct answer: C
Explanation
The clinical picture is compatible with primary adrenal failure. In a young person, Addison’s disease is the
most likely cause. The adrenocorticotropic hormone (ACTH) will be raised in the absence of negative
feedback due to cortisol deficiency.

Question 128 No Answer Given Incorrect

A 45-year-old man presented with a 2-year history of fatigue, 4-kg weight loss and pruritus. He also
complained of longstanding diarrhoea with the occasional passage of blood. He had drunk 20 units of
alcohol per week for many years.

On examination, he was mildly jaundiced.

Investigations:

serum albumin 38 g/L (37–49)


serum total bilirubin 95 µmol/L (1–22)
serum alanine aminotransferase 50 U/L (5–35)
serum alkaline phosphatase 315 U/L (45–105)
What is the most likely diagnosis?

Answers

A: alcoholic hepatitis
B: cholelithiasis
C: pancreatic cancer
D: primary biliary cirrhosis
E: primary sclerosing cholangitis

Correct answer: E
Explanation
The history suggests underlying undiagnosed ulcerative colitis, which is often associated with primary
sclerosing cholangitis (PSC), which tends to present with cholestatic derangement of liver function tests and
pruritus and fatigue. The alcohol history is not adequate to cause alcoholic hepatitis. Primary biliary cirrhosis
is much rarer in males. Cholelithiasis and pancreatic cancer are less likely with the history of painless jaundice
given.

Question 129 No Answer Given Incorrect

A 68-year-old man was advised to take isosorbide mononitrate 30 mg twice daily for angina pectoris.

What is the most important mechanism of action of this drug?

Answers

A: calcium channel blockade


B: increased cyclic adenosine monophosphate production
C: increased cyclic guanosine monophosphate production
D: potassium channel activation
E: sodium channel blockade

Correct answer: C
Explanation
Isosorbide mononitrate act as a nitric oxide (NO) donor, leading to stimulation of guanylate cyclase to
produce cyclic guanosine monophosphate (GMP), which mediates the vasodilatory effect of nitrates.

Question 130 No Answer Given Incorrect

A 25-year-old man presented with a 5-day history of foul-smelling diarrhoea, abdominal pain, nausea,
bloating and flatulence. He had returned from a holiday in Borneo 1 week previously.

On examination, he appeared dehydrated. Examination was otherwise normal.

Investigations:

stool microscopy motile trophozoites and cysts


What is the most likely causative organism?

Answers

A: Campylobacter jejuni
B: Cryptosporidium parvum
C: Escherichia coli O157
D: Giardia intestinalis
E: Shigella dysenteriae
Correct answer: D
Explanation
Although the commonest cause of infective diarrhoea is campylobacter, the history of small bowel
malabsorptive symptoms and of travel to an endemic area is suggestive of giardiasis. This is confirmed by the
finding of trophozoites and cysts on microscopy. Cryptosporidiosis is less common and would only show cysts
on microscopy.

Question 131 No Answer Given Incorrect

A 47-year-old man with a history of long QT syndrome presented with a 2-day history of a productive cough
and rigors.

He was found to have left lower lobe pneumonia.

Which antibiotic is most likely to further prolong his QT interval?

Answers

A: amoxicillin
B: ceftriaxone
C: clarithromycin
D: co-amoxiclav
E: doxycycline

Correct answer: C
Explanation
Antimicrobials that cause QT prolongation are: erythromycin, clarithromycin, moxifloxacin, fluconazole and
ketoconazole.

Doxycycline is an alternative therapy which does not cause QT prolongation so would be safe to use for this
patient.

Question 132 No Answer Given Incorrect

A 68-year-old man was taking long-term warfarin for atrial fibrillation and his INR was stable at 2.8.

Which coagulation factor is most likely to be present at a reduced activity?

Answers

A: factor V
B: factor VII
C: factor VIII
D: factor XII
E: fibrinogen

Correct answer: B
Explanation
Warfarin inhibits the vitamin K-dependent synthesis of clotting factors including factor II, VII, IX and X as well
as the regulatory factors protein C and protein S. Therefore, factor VII is the correct answer.

Question 133 No Answer Given Incorrect

A 53-year-old woman presented with a painful right ankle and was found to have a partial lower tibial
fracture.

Investigations:

serum creatinine 122 µmol/L (60–110)


serum corrected calcium 2.05 mmol/L (2.20–2.60)
serum phosphate 0.6 mmol/L (0.8–1.4)
serum alkaline phosphatase 180 U/L (45–105)
   
plasma parathyroid hormone 5.6 pmol/L (0.9–5.4)
serum 25-OH-cholecalciferol 32 nmol/L (>50)
What is the most appropriate treatment?

Answers

A: alendronic acid
B: calcium supplementation
C: colecalciferol
D: pamidronate disodium
E: phosphate supplementation

Correct answer: C
Explanation
This clinical picture is due to vitamin D deficiency leading to bone fracture. Treatment of choice is vitamin D
replacement.

Question 134 No Answer Given Incorrect

A 43-year-old woman presented with a 2-month history of progressive muscle weakness and shortness of
breath.
On examination, there was proximal limb weakness, and crackles at both lung bases.

Investigations:

ESR 55 mm/1st h (<20)


   
serum creatine kinase 3500 U/L (24–170)
What is the most likely diagnosis?

Answers

A: chronic inflammatory demyelinating polyneuropathy


B: inclusion body myositis
C: myasthenia gravis
D: polymyalgia rheumatica
E: polymyositis

Correct answer: E
Explanation
This is a classic description of polymyositis. There is a short history of progressive proximal muscle weakness
without sensory impairment and likely interstitial lung disease. Her serum creatine kinase (CK) is very high
with a high ESR showing an inflammatory cause. Polymyalgia should not be diagnosed in people below 50
years of age. It is not associated with such a high CK level either. Myasthenia gravis causes diurnal variation
in weakness. Demyelinating polyneuropathy is associated with sensory impairment. Inclusion body myositis is
seen in older age and is not associated with pneumonitis. Chronic inflammatory demyelinating
polyneuropathy, polymylagia rheumaica and myasthenia gravis do not cause such high CK levels.

Question 135 No Answer Given Incorrect

A 55-year-old woman presented with a 2-month history of lower back pain, for which she had been taking
paracetamol 4 g daily. She admitted to drinking 16 units of alcohol per week.

On examination, her BMI was 24.0 kg/m2 (18.0–25.0).

Investigations:

serum alanine aminotransferase 55 U/L (5–35)


serum alkaline phosphatase 90 U/L (45–105)
   
serum IgG 21.0 g/L (6.0–13.0)
serum IgA 2.4 g/L (0.8–3.0)
serum IgM 1.8 g/L (0.4–2.5)
What is the most likely cause of her abnormal liver function tests?

Answers

A: alcohol
B: autoimmune hepatitis
C: drug related
D: gallstones
E: primary biliary cirrhosis

Correct answer: B
Explanation
The raised IgG levels suggests autoimmune hepatitis and a diagnosis can be provisionally made while
awaiting the autoantibody profile. The alcohol consumption is not adequate to cause liver disease, nor is the
level of paracetamol use, which is rarely associated with chronic liver disease. Primary biliary cirrhosis is
associated with elevated IgM levels. There is no history of pain to suggest gallstones, which in any case
should not cause an elevation of IgG levels.

Question 136 No Answer Given Incorrect

A 56-year-old man presented with a 3-month history of cough and intermittent shortness of breath. He had
a 15 pack-year smoking history. For the past 2 years, he had worked in a bakery.

Investigations:

       lung function tests:

  pre-bronchodilator post-bronchodilator predicted


FEV1 3.00 L 3.52 L 3.60 L
FVC 4.55 L 4.60 L 4.55 L
FEV1/FVC ratio 0.66 0.77  
What is the most likely diagnosis?

Answers

A: chronic bronchitis
B: emphysema
C: extrinsic allergic alveolitis
D: occupational asthma
E: reactive airways dysfunction syndrome

Correct answer: D
Explanation
The clue is in the intermittent nature of his breathlessness and the fact that his symptoms started after
exposure to flour dust. He is a smoker, but the lung function tests are not consistent with emphysema or
chronic bronchitis as there is reversibility to bronchodilator which suggests asthma. Extrinsic allergic alveolitis
results as a result of exposure to allergens such as bird, fungi etc. and will usually result in a restrictive picture
on lung function.

Question 137 No Answer Given Incorrect

A 51-year-old man was referred for investigation of persistent isolated non-visible haematuria.

On examination, his BP was 121/76 mmHg. Urinalysis showed blood 2+.

Investigations:

serum creatinine 63 µmol/L (60–110)


eGFR (MDRD) >60 mL/min/1.73 m2 (>60)
   
ultrasound scan of abdomen two normal-sized, non-obstructed kidneys
What is the most appropriate next investigation?

Answers

A: ANCA antibodies
B: CT urography
C: cystoscopy
D: renal biopsy
E: serum IgA concentration

Correct answer: C
Explanation
Cystoscopy is correct as the guidelines suggest that cystoscopy in the over fifties should follow imaging of
the renal tract to look for urothelial malignancy (transitional cell carcinoma).

Question 138 No Answer Given Incorrect

A 54-year-old woman presented with a 3-hour history of severe anterior chest pain.

Investigations:

3-mm ST-segment depression in leads V1 to V3, with upright T


ECG
waves and tall R waves
A diagnosis of acute myocardial infarction was made and the patient was referred for primary percutaneous
coronary intervention.

Which coronary artery is most likely to be involved?

Answers

A: circumflex
B: first septal branch of the left anterior descending
C: left main stem
D: mid-left anterior descending
E: proximal left anterior descending

Correct answer: A
Explanation
This question is testing knowledge of coronary anatomy and corresponding ECG changes. The patient is
having a posterior ST-elevation myocardial infarction (STEMI), as noted by the fact there is a dominant R wave
in V1-3 and ST depression, which, if posterior leads were applied, would show ST elevation and Q waves in
V1-3 e.g. a mirror image. It is the circumflex that supplies the posterior surface of the heart, via the posterior
descending artery in around 15% of people, called a left dominant system; in 85% of people, the posterior
descending artery arises from the right coronary artery, called a right dominant system. Of the answers listed,
it is only a lesion in the circumflex that could produce these changes, the right coronary artery is not listed as
an option; therefore, the answer is circumflex. Of the other answers, first septal branch of the left anterior
descending would lead to ST elevation in I and aVL alone; left main stem would lead to widespread ST
elevation in most, if not all, leads (or possible ST elevation in aVR and ST depression in the anterior leads);
mid-left anterior descending and proximal left anterior descending would lead to anterior ST elevation, in V2-
6 and V1-4, respectively.

Question 139 No Answer Given Incorrect

A 42-year-old woman presented complaining of “worms” in the skin of her arms and legs for the past
9 months. She could feel the worms moving, and had tried to get them out of her skin using a needle. She
reported that the worms were gradually spreading and she was afraid they would lead to her death.

On examination, she was anxious. There were multiple needle marks on the skin of her arms and legs.
Examination was otherwise normal.

What is the most likely diagnosis?

Answers

A: delusional disorder
B: depressive psychosis
C: factitious disorder
D: generalised anxiety disorder
E: somatoform disorder

Correct answer: A
Explanation
Delusional parasitosis is a form of delusional disorder; the thought of death and some anxiety are secondary
and not the primary issue. Factitious disorder is the conscious production of symptoms for psychological gain,
which there is no evidence for in the question. Somatoform disorder, with physical symptoms unrelated or
out of proportion to underlying medical pathology, is a possible answer but the classic description and
intensity of belief around parasites makes delusional disorder correct.

Question 140 No Answer Given Incorrect

A 32-year-old man with chronic hepatitis C virus infection was being treated with directly acting antiviral
medication.

What investigation is most appropriate for assessing his sustained response to treatment?

Answers

A: anti-hepatitis C antibody concentration


B: HCV viral load
C: liver biopsy
D: serum alanine aminotransferase
E: serum CRP concentration

Correct answer: B
Explanation
Virological response using HCV viral load.

Question 141 No Answer Given Incorrect

A 56-year-old man presented with a red and swollen third toe on return from a skiing holiday. It was not
particularly painful and he was otherwise well. He had a 15-year history of type 2 diabetes mellitus, which
had been treated with metformin for several years, and plaque psoriasis treated with topical agents.

On examination, there was a small discharging ulcer at the tip of the toe and reduced vibration sensation in
both feet.

Investigations: 

eGFR (MDRD) 55 mL/min/1.73 m2 (>60)


serum CRP 21 mg/L (<10)
What is the most likely diagnosis?

Answers

A: Charcot neuroarthropathy
B: chilblains (pernio)
C: gout
D: osteomyelitis
E: psoriatic dactylitis

Correct answer: D
Explanation
This is a longstanding type 2 diabetes mellitus complicated by peripheral neuropathy. An injury was sustained
which led to an infected ulcer of the toe. A red swollen toe is indicative of osteomyelitis.

Question 142 No Answer Given Incorrect

A 37-year-old woman presented with a 1-day history of abdominal pain and diarrhoea.

On examination, she was drowsy and hallucinating, and had widespread purpura.

Investigations:

haemoglobin 92 g/L (115–165)


white cell count 13.3 × 109/L (4.0–11.0)
platelet count 17 × 109/L (150–400)
   
serum creatinine 195 µmol/L (60–110)
What abnormality is most likely to be present on blood film examination?

Answers

A: acanthocytes
B: elliptocytes
C: fragmented cells
D: stomatocytes
E: target cells

Correct answer: C
Explanation
Microangiopathic anaemia and thrombocytopenia are the essential features of thrombotic thrombocytopenic
purpura (TTP). The presence of fragmented red cells is an important feature of microangiopathic anaemia and
is therefore the correct answer. While impaired renal function is in the pentad of TTP, a lot of patients with
bona fide TTP do not have renal impairment.

Question 143 No Answer Given Incorrect

An 87-year-old woman was admitted with increasing drowsiness, which resulted in her not eating or
drinking. She had a history of dementia and was living in a nursing home.

Investigations:

serum sodium 178 mmol/L (137–144)


serum potassium 6.0 mmol/L (3.5–4.9)
serum urea 18.0 mmol/L (2.5–7.0)
serum creatinine 190 µmol/L (60–110)
fasting plasma glucose 4.5 mmol/L (3.0–6.0)
What measurement corresponds most closely to her serum osmolality?

Answers

A: 210 mosmol/kg
B: 310 mosmol/kg
C: 350 mosmol/kg
D: 390 mosmol/kg
E: 410 mosmol/kg

Correct answer: D
Explanation
Calculated osmolality = (2 × Na) + (2 × K) + glucose + urea (all in mmol/L).

Question 144 No Answer Given Incorrect

A 32-year-old man presented with increasing headache and facial weakness. Two weeks previously, he had
developed a right-sided facial palsy with pain behind the right ear. The previous summer, he had camped
with his family in southern Germany.

Examination revealed mild photophobia, neck stiffness, and a right lower motor neurone facial weakness, but
no other neurological signs.

Which organism is the most likely cause?

Answers
A: Borrelia burgdorferi
B: Brucella abortus
C: Cryptococcus neoformans
D: Listeria monocytogenes
E: Neisseria meningitidis

Correct answer: A
Explanation
The clues for Lyme disease are apparent: travel to relevant area, followed by lower motor neurone VII nerve
palsy, all of which would be unusual; with other distractors. Listeria and cryptococcus are unlikely in a young
immunocompetent man in the UK.

Question 145 No Answer Given Incorrect

A 76-year-old woman presented with a 6-month history of urinary urgency, nocturia and urge incontinence.
She was taking immediate-release oxybutynin but this was discontinued after 6 weeks because of lack of
significant improvement and severe mouth dryness.

Rectal and vaginal examinations were normal.

Investigations:

midstream urine culture no bacterial growth


post-void residual urine volume 40 mL
What is the most appropriate next treatment?

Answers

A: desmopressin
B: duloxetine
C: flavoxate
D: propantheline
E: solifenacin

Correct answer: E
Explanation
The patient has overactive bladder. If oxybutynin is ineffective, another antimuscarinic drug, like solifenacin,
should be used.

Question 146 No Answer Given Incorrect

A 68-year-old woman presented with a 3-day history of discomfort when brushing her hair, pain on chewing
and an episode of blurred vision.
A blood test was performed to check ESR and CRP.

What is the most important next step?

Answers

A: CT scan of head
B: measure ANCA
C: prescribe diclofenac
D: prescribe prednisolone
E: temporal artery biopsy

Correct answer: D
Explanation
This question describes a classic case of giant cell arteritis. Patient’s sex and age group are appropriate for the
diagnosis. She has rather short history of scalp irritation with jaw claudication and impending vision loss. This
is a rheumatological emergency. Time is of the essence and hence corticosteroid therapy should be started
immediately. Temporal artery biopsy can give a definitive diagnosis but logistically takes longer to organise.
Treatment should not be delayed for this procedure. A CT scan of head and ANCA test are not going to
support the diagnosis. Diclofenac has no role in management of this condition.

Question 147 No Answer Given Incorrect

A 72-year-old woman presented with progressive tiredness over the past few months and was found to have
iron deficiency anaemia. She had osteoarthritis and was regularly taking ibuprofen and paracetamol.

Investigations:

upper
1-cm duodenal ulcer with no evidence of recent
gastrointestinal
bleeding; Helicobacter pylori negative
endoscopy
She was treated with omeprazole.

What is the most appropriate next investigation?

Answers

A: colonoscopy
B: CT scan of abdomen
C: no further investigation needed
D: repeat upper gastrointestinal endoscopy in 8 weeks
E: video capsule endoscopy

Correct answer: A
Explanation
Iron deficiency anaemia (IDA) in a 72-year-old can be caused by underlying malignancy in up to 10% of
cases. This is often right-sided colonic cancers, which are otherwise asymptomatic and, therefore, a
colonoscopy is almost always required in such cases. Repeat endoscopy is not required to assess healing of
duodenal ulcer. CT scan is less sensitive than colonoscopy in identifying colonic pathology and should only be
considered if there are contraindications to colonoscopy. Video capsule endoscopy is reserved for cases of
difficult-to-diagnose IDA following upper gastrointestinal endoscopy/colonoscopy.

Question 148 No Answer Given Incorrect

A 58-year-old woman with autosomal dominant polycystic kidney disease presented with severe right-sided
flank pain.

On examination, she was in distress. Her temperature was 37.4°C, and the right kidney was palpable and
tender. Urinalysis showed blood 1+, protein 1+.

Investigations:

white cell count 8.2 × 109/L (4.0–11.0)


What is the most likely reason for her pain?

Answers

A: cyst haemorrhage
B: cyst infection
C: cyst torsion
D: renal calculus
E: renal carcinoma

Correct answer: A
Explanation
Cyst haemorrhage is correct as the most likely cause of loin pain as complication of autosomal dominant
polycystic kidney disease without infection. The white cell count is normal suggesting no infection, and no
visible haematuria, suggesting it is unlikely to be renal calculus or carcinoma.

Question 149 No Answer Given Incorrect

What is the site of origin of circulating B-type natriuretic peptide in patients with chronic heart failure?

Answers

A: cardiac atria
B: cardiac ventricles
C: hypothalamus
D: juxtaglomerular apparatus
E: posterior pituitary gland

Correct answer: B
Explanation
B-type natriuretic peptide is released from myocytes in the cardiac ventricles in response to stretch-associated
high ventricular filling pressures.

Question 150 No Answer Given Incorrect

A 57-year-old woman was found to have cervical cancer and was treated with surgery, followed by
chemotherapy with cisplatin.

What is the most likely adverse effect of this treatment?

Answers

A: acute hepatitis
B: cardiomyopathy
C: hypocalcaemia
D: pulmonary fibrosis
E: sensory neuropathy

Correct answer: E
Explanation
Recognised adverse effects of platinum compounds include nephrotoxicity and neurotoxicity which manifests
as sensory neuropathy or hearing loss.

Question 151 No Answer Given Incorrect

A 52-year-old man presented as an emergency with shortness of breath and leg swelling. He had a history of
hypertension, hyperlipidaemia and chronic alcohol misuse. Congestive heart failure was diagnosed, and he
was treated with furosemide, bisoprolol and enalapril.

On examination, his JVP was elevated and there was a pansystolic murmur with a third heart sound.

Investigations following treatment:

sinus rhythm at 60 beats/min with left bundle


ECG
branch block
   
transthoracic left ventricular ejection fraction 25% with global
echocardiogram hypokinesis
What is the most appropriate additional pharmacological treatment?

Answers

A: digoxin
B: eplerenone
C: hydralazine
D: isosorbide dinitrate
E: ivabradine

Correct answer: B
Explanation
This question is testing knowledge on the medical management of heart failure. The patient has an acute
presentation of heart failure, likely biventricular systolic dysfunction (also known as congestive cardiac
failure), given the presentation of left- (breathlessness) and right- (peripheral oedema) sided symptoms and
the history of alcohol and high BP. He is started on treatment with a beta-adrenoceptor blocker and ACE
inhibitor for morbidity and mortality benefit and then a diuretic for symptom control. The question is then,
following investigations, what additional therapy would be indicated and, therefore, offer additional benefit.
The patient is in sinus rhythm at 60 beats/min, so digoxin, which should be offered third line in worsening or
severe heart failure (not second line as in this case), is not indicated. Eplerenone is the correct answer, as an
aldosterone antagonist should be offered second line in patients with moderate to severe heart failure. The
patient’s BP is not given, nor is any intolerance to the ACE inhibitor suggested, so offering an arterial
dilator (hydralazine) or a venodilator (isosorbide mononitrate), which are often given in combination,
particularly in African-Caribbean patients, as first or second line, is not indicated. Even though left ventricular
ejection fraction less than 35% is an indication for ivabradine, it is not indicated because the patient’s heart
rate is not above 75 beats/min, it is not following 4 weeks after starting first- and second-line treatments,
and the patient does not have ischaemic heart disease or require an additional antianginal.

Question 152 No Answer Given Incorrect

A 47-year-old man presented with episodes of feeling that he was about to have a heart attack, associated
with palpitations and sweating. He recovered fully within 30 minutes. He had experienced 12 episodes over
the previous 4 weeks. He had been working long hours and was concerned about a forthcoming interview
for a new job.

Examination was normal.

What is the most likely diagnosis?

Answers

A: acute stress reaction


B: adjustment disorder
C: generalised anxiety disorder
D: hypochondriasis
E: panic disorder

Correct answer: E
Explanation
This is a typical description of panic disorder: episodic acute anxiety with associated physical symptoms.
Symptoms are not present outside of attack, which would be characteristic of generalised anxiety disorder.
Acute stress reaction needs a major acute stressor, normally terrifying, and is associated with generalised
anxiety and arousal, sleep disturbance, possible hyper-vigilance and avoidance; the patient may go on to
develop post-traumatic stress disorder. Adjustment reaction again needs an event and would be associated
with more generalised anxiety, sadness, worry, low mood and tearfulness. Hypochondriasis is excessive
concern and worry about serious physical illness.

Question 153 No Answer Given Incorrect

A 52-year-old woman presented with a 6-month history of upper limb weakness that had progressed to
affect her lower limbs. Over the previous 2 months, she had been complaining of increasing dysphagia to
liquids more than solids, and worsening shortness of breath at rest.

On examination, she had a brisk jaw jerk, tongue fasciculations and MRC grade 4 muscle power in shoulder
abduction and hip flexion bilaterally.

What is the most likely diagnosis?

Answers

A: motor neurone disease


B: multiple sclerosis
C: myasthenia gravis
D: spinobulbar muscular atrophy
E: syringobulbia

Correct answer: A
Explanation
The short, progressive, purely motor history with upper motor neurone (UMN) and lower motor neurone
(LMN) signs is characteristic of motor neurone disease (MND)/amyotrophic lateral sclerosis (ALS). Multiple
sclerosis would not cause LMN signs (tongue fasciculation), myasthenia gravis can mimic MND but would not
cause the signs and tends to be fatigable not progressive, spinobulbar muscular atrophy usually has a much
longer history and LMN only, and syringobulbia would most likely produce sensory involvement.

Question 154 No Answer Given Incorrect


A 51-year-old man presented with a 2-month history of intermittent breathlessness, and a 1-year history of a
blocked nose. He had been found to have angina 2 years previously and was taking aspirin, ramipril,
simvastatin, atenolol and omeprazole.

Investigations:

FEV1 2.5 L (75% predicted)


FVC 4.0 L (88% predicted)
Which drug is most likely to be causing his symptoms?

Answers

A: aspirin
B: atenolol
C: omeprazole
D: ramipril
E: simvastatin

Correct answer: A
Explanation
Aspirin can worsen asthma and cause nasal symptoms in a sub-group of patients with asthma. Aspirin is
associated with nasal polyps, hence the description of a blocked nose. Ramipril and other ACE inhibitors can
cause a cough and atenolol can cause breathlessness but neither of these is associated with nasal polyps.
Omeprazole can be used to treat reflux-related cough.

Question 155 No Answer Given Incorrect

A 17-year-old girl was informed at a blood donation session that she was not able to donate blood. She was
a carrier of sickle haemoglobin. She had had a pelvic fracture following a road traffic collision the previous
year, for which she had required a blood transfusion. She had been on holiday in Kenya 3 years previously.
She did not smoke, drink or use recreational drugs.

On examination, she had a tattoo on her arm. She weighed 63 kg.

What is the most likely reason for her not being able to donate blood?

Answers

A: history of travel to a malarial area


B: previous blood transfusion
C: sickle haemoglobin carrier
D: tattoo
E: weight
Correct answer: B
Explanation
Of the options, people who have been transfused will not be accepted as blood donors, in order to reduce
the risk of transmitting variant Creutzfeldt–Jakob disease to recipients
(https://my.blood.co.uk/KnowledgeBase/index/transfusion). 

Question 156 No Answer Given Incorrect

The Health Survey for England recorded the age, sex, ethnicity, height, weight, smoking status, alcohol
consumption and systolic BP in a representative sample of the population.

What is the best graphical method for illustrating the relation between age and BP?

Answers

A: block diagram
B: forest plot
C: histogram
D: pie chart
E: scatter plot

Correct answer: E
Explanation
A scatter plot facilitates charting of two variables allowing any correlation to be calculated.

Question 157 No Answer Given Incorrect

A 60-year-old woman with advanced metastatic cancer was treated with subcutaneous haloperidol infusion.

What is the main site of action of this antiemetic?

Answers

A: area postrema
B: cerebral cortex
C: gut wall
D: substantia nigra
E: vestibular nuclei

Correct answer: A
Explanation
Haloperidol is a D2-receptor antagonist. It has a central antiemetic action binding to receptors in the area
postrema, which is a medullary structure controlling vomiting.
Question 158 No Answer Given Incorrect

A 72-year-old man became suddenly short of breath during his first dialysis session via a dialysis catheter.

On examination, he had facial oedema, red eyes and a blanching rash on his arms and legs. His pulse was
118 beats per minute and his BP was 108/72 mmHg. His oxygen saturation was 96% (94–98) breathing air.

What is the most likely immunological diagnosis?

Answers

A: C1-esterase deficiency
B: hypocomplementaemic urticarial vasculitis
C: mast cell degranulation syndrome
D: type 1 hypersensitivity reaction
E: type 3 hypersensitivity reaction

Correct answer: D
Explanation
Type 1 hypersensitivity reaction is the correct answer. It is a rare but known complication of dialysis
membranes, but the question pertains to type 1 hypersensitivity reactions rather than being specific for
dialysis.

Question 159 No Answer Given Incorrect

A 76-year-old man presented with a 3-year history of a gradually enlarging, red plaque on his leg. He was an
ex-serviceman who had worked in the Mediterranean for several years.

On examination, there was an erythematous scaly plaque, 5 × 6 cm in diameter, on his shin. The edge was
well demarcated.

What is the most likely diagnosis?

Answers

A: Bowen’s disease
B: discoid eczema
C: pityriasis rosea
D: psoriasis vulgaris
E: secondary syphilis

Correct answer: A
Explanation
The combination of a history of working in the Mediterranean for several years (indicating significant
previous sun exposure) and the patient’s age suggests that this may be a sun-induced lesion resulting from
prior UV radiation damage. The site of the lesion on the shin would be compatible with Bowen’s disease,
discoid eczema or psoriasis vulgaris, but the 3-year history of a gradually enlarging lesion at that site and the
fact that there is no mention of a rash or lesions elsewhere makes Bowen’s disease the most likely diagnosis.

Question 160 No Answer Given Incorrect

A 27-year-old man presented with a 1-day history of fever and a swollen right leg. Three years previously, he
had developed an urticarial rash, tongue swelling and stridor after taking amoxicillin.

On examination, he appeared extremely unwell. His temperature was 39.8°C, his pulse was 130 beats per
minute and his BP was 66/38 mmHg. He had severe cellulitis of the right leg.

What is the most appropriate empirical treatment?

Answers

A: ceftriaxone
B: clindamycin
C: gentamicin
D: meropenem
E: piperacillin with tazobactam

Correct answer: B
Explanation
This is a patient with a history suggestive of anaphylaxis to a penicillin. Therefore, beta-lactam drugs should
be avoided. This excludes ceftriaxone, meropenem and piperacillin with tazobactam. Clindamycin is a
macrolide antibiotic and, therefore, is the most appropriate. Gentamicin should be reserved for severely
unwell patients and is usually used in combination. 

Question 161 No Answer Given Incorrect

A 67-year-old man presented with a 6-month history of muscle rigidity and a resting tremor. A diagnosis of
Parkinson’s disease was made and he was treated with ropinirole.

What is the mechanism of action of ropinirole?

Answers

A: catechol-O-methyltransferase inhibitor
B: DOPA-decarboxylase inhibitor
C: dopamine receptor agonist
D: dopamine transporter inhibitor
E: monoamine oxidase B inhibitor

Correct answer: C
Explanation
Ropinirole is an agonist at dopamine receptors and is used as an adjunct to levodopa to treat Parkinson’s
disease.

Question 162 No Answer Given Incorrect

A 55-year-old woman presented with vomiting and abdominal pain. She had a 3-month history of thirst and
polyuria.

On examination, her pulse was 90 beats per minute and her BP was 90/65 mmHg. There was reduced skin
turgor.

Investigations:

serum sodium 150 mmol/L (137–144)


serum potassium 3.0 mmol/L (3.5–4.9)
serum corrected calcium 3.40 mmol/L (2.20–2.60)
serum phosphate 0.5 mmol/L (0.8–1.4)
   
plasma parathyroid hormone 6.1 pmol/L (0.9–5.4)
What is the most likely cause of her hypophosphataemia?

Answers

A: decreased gastrointestinal absorption


B: increased bone resorption
C: intracellular shift of phosphate
D: reduced renal reabsorption of phosphate
E: vomiting

Correct answer: D
Explanation
This patient has symptomatic hypercalcaemia with low serum phosphate and slightly elevated plasma
parathyroid hormone (PTH).

Decreased gastrointestinal absorption could be a cause of this but it is not the most likely cause.

Bone resorption releases bound phosphate, so will not cause hypophosphataemia.


There is no mention of any actions which could precipitate re-feeding syndrome and, therefore, intracellular
shift of phosphate.

The phosphate content of vomitus is insufficient to be the most likely cause of this result.

Reduced renal absorption of phosphate due to raised PTH is, therefore, the most likely cause of her
hypophosphataemia.

Question 163 No Answer Given Incorrect

A 72-year-old man presented with a 6-week history of diffuse pain in the right arm, and numbness in the
right thumb.

On examination, there was a reduced right biceps reflex and sensory loss in the right thumb.

What is the most likely diagnosis?

Answers

A: C6 radiculopathy
B: carpal tunnel syndrome
C: Pancoast tumour
D: thoracic outlet (cervical rib) syndrome
E: ulnar nerve compression at the elbow

Correct answer: A
Explanation
The history and exam are suggestive of a C6 radiculopathy, although the biceps reflex is mainly served by C5,
and less so by C6; its dermatome does not extend to the hand. Carpal tunnel syndrome affects the median
nerve, so would not cause reduced biceps (C5/6) reflex loss, ditto ulnar compression, and the sensory loss is
wrong. Pancoast tumour typically involves lower roots, and thoracic outlet syndrome is rarely so focal.

Question 164 No Answer Given Incorrect

A 43-year-old woman presented with a 6-week history of pain and swelling of her fingers and toes. She had
had a self-limiting episode of guttate psoriasis 15 years previously. Her older sister had systemic lupus
erythematosus.

On examination, the patient had diffuse swelling of her right index and left ring fingers, and her left fourth
toe.

Investigations:
ESR 18 mm/1st h (<20)
   
antinuclear antibodies 1:60 dilution (negative at 1:20)
rheumatoid factor 10 kIU/L (<30)
What is the most likely diagnosis?

Answers

A: osteoarthritis
B: psoriatic arthritis
C: rheumatoid arthritis
D: systemic lupus erythematosus
E: systemic sclerosis

Correct answer: B
Explanation
The diagnosis here is psoriatic arthritis. This woman has history of psoriasis and clinical examination describes
dactylitis of fingers and toes in an asymmetric fashion. ESR may not be high in these circumstances and mild
elevation of antinuclear antibodies is of no great significance. These test results work here as confounders.
Diagnosis of osteoarthritis is unusual in this age group in this pattern. Rheumatoid arthritis presents with
symmetrical small joint inflammation and does not cause dactylitis. There are no clinical features to support a
diagnosis of systemic lupus erythematosus or systemic sclerosis in this scenario.

Question 165 No Answer Given Incorrect

A 36-year-old woman presented for review of her ventricular septal defect. She was considering pregnancy.
An echocardiogram was performed.

What finding would indicate the highest potential risk for a subsequent pregnancy?

Answers

A: associated aortic regurgitation


B: associated bicuspid aortic valve
C: high systolic pressure gradient between left and right ventricles
D: pulmonary hypertension
E: ventricular septal defect measuring half the aortic root diameter

Correct answer: D
Explanation
This question is testing knowledge of the risks of cardiac diseases in association with pregnancy. Regurgitant
valvular lesions, left or right sided, are well tolerated in pregnancy, so associated aortic regurgitation  is
incorrect. An associated bicuspid valve with a ventricular septal defect (VSD) would be unusual, but provided
the valve function is preserved and not stenotic (stenotic lesions are not well tolerated in pregnancy), then
this would not cause problems and so, as you are not told the gradient across the valve or whether it is
stenotic, associated bicuspid aortic valve is incorrect. A high systolic pressure gradient between the left and
right ventricles is physiologically normal and what causes a left to right shunt in a VSD, and so high systolic
pressure gradient between left and right ventricles is incorrect. In Eisenmenger syndrome, where the pressure
in the right is higher than the left, this causes a right to left shunt and would increase risk for a subsequent
pregnancy. Pulmonary hypertension significantly increases the risk to both mother and baby during
pregnancy and so is the correct answer. Finally, VSD measuring half the aortic diameter is irrelevant, as you
are not told the size of the aortic root diameter in the first instance. Assuming it is the normal size (2.5 cm
approximately) which is usually normalised for body surface area, while it would be large, it is the
haemodynamic effects which would be important e.g. leading to left ventricular failure or pulmonary
hypertension, but again these are not listed.

Question 166 No Answer Given Incorrect

A study evaluated the effectiveness of a new analgesic in patients with painful knee osteoarthritis. Subjects
were asked to assess their pain on a 10-point scale both before and after 6 weeks of treatment.

What is the most appropriate test to ascertain whether the treatment reduced the pain?

Answers

A: chi-squared test
B: Mann–Whitney U test
C: one-sample t-test
D: two-sample t-test
E: Wilcoxon signed-rank test

Correct answer: E
Explanation
These are not independent variables, and the size of the group is not mentioned, so it cannot be assumed to
be a normal distribution. Wilcoxon signed-rank test is, therefore, the correct answer as it is a non-parametric
used to compare related data groups.

Question 167 No Answer Given Incorrect

A 72-year-old man presented with loss of the lower half of the visual field of his right eye, with no associated
pain. He had type 2 diabetes mellitus and was being treated for hypertension.

Examination showed normal visual acuity in both eyes, with an inferior altitudinal field defect in the right eye.
Fundoscopy of the left eye was normal, and the upper part of the right optic disc showed mild disc swelling.
No fundal haemorrhages were seen. His BP was 160/90 mmHg.

What is the most likely diagnosis?


Answers

A: optic neuritis
B: panuveitis
C: posterior communicating artery aneurysm
D: retinal artery occlusion
E: retinal vein thrombosis

Correct answer: D
Explanation
The visual field defect indicates a likely vascular cause. Branch retinal vein thrombosis is typically associated
with fundal haemorrhages, not seen in this case, making an arterial explanation more likely. Optic neuritis
and panuveitis would usually lead to impairment of visual acuity, often with pain. A posterior communicating
artery aneurysm more typically presents with a pupillary involving III nerve palsy.

Question 168 No Answer Given Incorrect

A 68-year-old man was referred with muscle contractures following a traumatic head injury 3 years
previously.

On examination, there was a left-sided torticollis.

What muscle is most likely to be responsible?

Answers

A: left omohyoid
B: left sternocleidomastoid
C: left trapezius
D: right omohyoid
E: right sternocleidomastoid

Correct answer: E
Explanation
Torticollis is caused by contracture of the contralateral sternocleidomastoid muscle.

Question 169 No Answer Given Incorrect

A 20-year-old man presented with a 24-hour history of drooling, a hoarse voice, sore throat and difficulty in
breathing.

On examination, he had audible stridor.


Investigations:

lateral X-ray of neck features of acute epiglottitis


What is the most likely causative organism?

Answers

A: Haemophilus influenzae
B: Klebsiella pneumoniae
C: Neisseria meningitidis
D: Pseudomonas aeruginosa
E: Streptococcus pneumoniae

Correct answer: A
Explanation
Haemophilus influenzae is the most common cause of acute epiglottitis, especially in younger patients. Its
incidence is reducing due to vaccination.

Question 170 No Answer Given Incorrect

What cellular component contains double-stranded circular DNA?

Answers

A: Golgi apparatus
B: mitochondrion
C: nucleus
D: peroxisome
E: proteasome

Correct answer: B
Explanation
Mitochondrial DNA is matrilineal and is in a covalently bonded circular form. Maternally inherited diseases are
associated with mutations in this DNA.

Question 171 No Answer Given Incorrect

An 85-year-old woman presented with breathlessness that had developed over the previous 2 years. She
complained of a cough that frequently woke her from sleep. She had been treated with radiotherapy for a
cervical lymphoma 20 years previously.

On examination, she was thin, but had no abnormal physical signs. Her oxygen saturation was 95% (94–98)
breathing air.
Investigations:

high-resolution basal fibrosis associated with dilated bronchi, but no


CT scan of chest honeycombing or ground-glass change
What is the most likely diagnosis?

Answers

A: bronchiectasis
B: chronic aspiration
C: radiation fibrosis
D: sarcoidosis
E: usual interstitial pneumonia-type pulmonary fibrosis

Correct answer: B
Explanation
The symptoms described are consistent with chronic aspiration occurring in the elderly. The CT findings are
radiological bronchiectasis, a sequela of recurrent episodes of aspiration pneumonia. This does not, however,
equate to a clinical diagnosis of bronchiectasis.

The absence of honeycombing and ground-glass opacities counts against the diagnosis of radiation fibrosis
and usual interstitial pneumonia-type pulmonary fibrosis.

Question 172 No Answer Given Incorrect

A 24-year-old man presented with a bluish discoloration of his body, lips and nails. He denied any past
medical history.

On examination, he had central cyanosis and a steel-grey complexion. There were no other abnormal
findings. 

Investigations:

haemoglobin 172 g/L (130–180)


   
arterial blood gases, breathing air:  
PO2 13.6 kPa (11.3–12.6)
oxygen saturation 90% (94–98)
What is the most likely diagnosis?

Answers

A: cyanotic congenital heart disease


B: hepatic cirrhosis
C: methaemoglobinaemia
D: primary proliferative polycythemia
E: pulmonary arteriovenous shunt

Correct answer: C
Explanation
The investigations in this young patient demonstrate the elevated PO2 in combination with the reduced
oxygen saturation. This would be in keeping with a left shift of the oxygen dissociation curve where there is
increased affinity of binding of oxygen with decreased release to the tissues. A well-recognised cause of this
is methaemoglobinaemia, either acquired or congenital. Cyanotic congenital heart disease and pulmonary
arteriovenous shunt would more typically be associated with a more marked elevation of haemoglobin and
reduced oxygen saturation. Primary proliferative polycythaemia should not be associated with hypoxia.
Hepatic cirrhosis is not indicated by the findings or history.

Question 173 No Answer Given Incorrect

A 50-year-old woman with discoid lupus erythematosus presented with an apparent worsening of her rash
despite treatment with topical corticosteroids and photoprotection.

On examination, there were discrete erythematous, scaly plaques with follicular plugging.

What additional treatment is most appropriate?

Answers

A: hydroxychloroquine
B: isotretinoin
C: nicotinamide
D: oxytetracycline
E: quinine sulfate

Correct answer: A
Explanation
We are told that this patient has discoid lupus erythematosus and that her rash has worsened despite first-
line treatments with topical corticosteroids (admittedly, we are not told the potency of the corticosteroids)
and photoprotection (we have to presume that the patient used good photoprotection which included
sunscreens protecting against UVB and UVA, as well as appropriate clothing and staying out of sunlight
when possible). Discoid lupus erythematosus can cause scarring, and frequently affects the face, so it is
important to treat the condition adequately in order to prevent scarring. Antimalarials such as
hydroxychloroquine are recommended as first-line and long-term systemic treatment in skin lupus patients
with a high risk of scarring (e.g. European Dermatology Forum/European Academy of Dermatology and
Venereology guidelines JEADV 2017, 31, 389–404), therefore hydroxychloroquine is the most suitable
treatment from the above list. In addition, a Cochrane systematic review update in 2017 indicated that, while
hydroxychloroquine and acitretin (which is not in the above list) appear to be of equal efficacy in terms of
complete resolution, adverse effects might be more frequent with acitretin (which is teratogenic in women),
thus supporting the use of hydroxychloroquine in the above case. Note that although quinine sulfate can be
used to treat malaria and was used many years ago for discoid lupus erythematosus, this drug has been
replaced by synthetic antimalarials (such as hydroxychloroquine) for treatment of discoid lupus
erythematosus. Furthermore, quinine can cause photosensitivity as an adverse effect.

Question 174 No Answer Given Incorrect

A 69-year-old man presented to the outpatient clinic accompanied by his wife. She complained of disturbed
sleep and said that her husband had become aggressive during sleep. She reported that he lashed out and
screamed, and he had occasionally kicked her and fallen out of bed. These episodes could occur up to four
times per night. On waking, he had vivid recall of the events but seemed unable to prevent them from
happening.

On examination, his BMI was 32.0 kg/m2 (18.0–25.0).

What is the most likely diagnosis?

Answers

A: nocturnal frontal lobe epilepsy


B: non-epileptic attack disorder
C: periodic limb movement disorder
D: rapid eye movement sleep behaviour disorder
E: sleep apnoea

Correct answer: D
Explanation
This is a text-book description of rapid eye movement (REM) sleep behavioural disturbance, with patients
acting out their dreams. Patients are usually easily roused from this, unlike non-REM parasomnias. Sleep
apnoea is characterised by snoring and apnoeic episodes, non-epileptic attack disorder rarely arises from true
sleep, and periodic limb movement disorder is rarely aggressive, although can disturb bed partners. Frontal
lobe attacks can be difficult to distinguish, but are rarely associated with dreaming, rarely involve targeted
violence, and are less frequent.

Question 175 No Answer Given Incorrect

A 63-year-old man presented with dyspnoea and left basal chest pain over the past 2–3 hours. His symptoms
were aggravated by coughing and movement. He smoked 10 cigarettes per day and had undergone
treatment for thrombosis of the left femoral vein 18 months previously.
On examination, his pulse was 110 beats per minute and regular. He had high-pitched polyphonic wheezes
and late inspiratory crackles at the left lung base. There was oedema of both calves, more marked on the
left.

Investigations:

arterial blood gases, breathing air:  


PO2 9.5 kPa (11.3–12.6)
PCO2 4.9 kPa (4.7–6.0)
pH 7.40 (7.35–7.45)
H+ 40 nmol/L (35–45)
bicarbonate 22 mmol/L (21–29)
   
chest X-ray minor left basal atelectasis
What is the most appropriate diagnostic investigation?

Answers

A: CT pulmonary angiography
B: D-dimer concentration
C: echocardiography
D: ultrasound scan of lower limbs and pelvis
E: ventilation/perfusion isotope lung scan

Correct answer: A
Explanation
The clinical features described are strongly suggestive of acute pulmonary embolism: acute dyspnoea and
focal pleuritic chest pain, with risk factors for venous thromboembolism, namely a previous deep venous
thrombosis (DVT) episode and cigarette smoking.

The most appropriate investigation would be a confirmatory test for pulmonary embolism, of which CT
pulmonary angiography performs better than ventilation/perfusion isotope lung scan.

D-dimer concentration is not helpful in this situation as the pre-test probability is very high. Echocardiography
will help to ascertain the degree of right ventricular dysfunction, but in itself is not diagnostic of pulmonary
embolism.

The presence of a concomitant left lower limb DVT is suggested in the stem, but while an ultrasound scan of
lower limbs and pelvis is useful to determine that, the focus should first be on confirming the pulmonary
embolism.

Question 176 No Answer Given Incorrect


A 54-year-old man with type 2 diabetes mellitus attended for clinic review. He was found to have a BP of
160/98 mmHg. His medication comprised ramipril 5 mg daily, gliclazide 80 mg daily, bendroflumethiazide
5 mg daily, and metformin 500 mg three times daily.

Investigations:

serum creatinine 225 µmol/L (60–110)


eGFR 28 mL/min/1.73 m2 (>60)
haemoglobin A1c 69 mmol/mol (20–42)
   
serum cholesterol 8.3 mmol/L (<5.2)
What is the most appropriate next step in his management?

Answers

A: increase ramipril
B: start atenolol
C: start insulin
D: stop bendroflumethiazide
E: stop metformin

Correct answer: E
Explanation
Stopping metformin remains the first step in diabetic patients with serum creatinine over 200 µmol/L.
Ramipril may be increased later and he will need better glycaemic control, but these are not immediate steps.

Question 177 No Answer Given Incorrect

A 65-year-old man, who was undergoing investigations for tinnitus, was found to have a small intrasellar
pituitary lesion measuring 0.9 cm.

Investigations showed no evidence of pituitary hormone excess. A repeat MR scan of brain 1 year later
showed no change.

What is the most likely clinical consequence of this lesion?

Answers

A: anterior hypopituitarism
B: bitemporal hemianopia
C: none
D: pituitary apoplexy
E: posterior hypopituitarism
Correct answer: C
Explanation
This is a nonfunctioning pituitary lesion which has not changed in size and does not lead to any clinical
complication.

Question 178 No Answer Given Incorrect

A 57-year-old woman with longstanding rheumatoid arthritis presented with a 5-day history of severe pain in
her right eye. The pain radiated to the frontal region and face, and was worse at night.

On examination, the right eye was red, with normal visual acuity and fundal appearance.

What is the most likely cause of her pain?

Answers

A: acute anterior uveitis


B: acute glaucoma
C: conjunctivitis
D: scleritis
E: subconjunctival haemorrhage

Correct answer: D
Explanation
Scleritis/episcleritis is due to primary ocular disease in about 50% of cases. In the other half, it is secondary to
various disorders, of which the autoimmune inflammatory disorders are most common, particularly
rheumatoid arthritis (RA). Conjunctivitis is not usually so painful and typically bilateral. Subconjunctival
haemorrhage always looks impressive but usually asymptomatic, and glaucoma usually affects acuity,
although this would be an important differential. Anterior uveitis could also mimic this appearance, but the
underlying RA makes episcleritis more likely.

Question 179 No Answer Given Incorrect

A 44-year-old man presented with a 2-month history of sores in his mouth and nose, and a 10-day history of
blisters on his scalp and back.

On examination, there were flaccid bullae and erosions on his scalp and back. There were erosions of the oral
and nasal mucosae.

Pending results of skin biopsy, what is the most appropriate empirical treatment?

Answers
A: aciclovir
B: itraconazole
C: no treatment required
D: phenoxymethylpenicillin
E: prednisolone

Correct answer: E
Explanation
The combination of flaccid bullae (i.e. large blisters) and erosions on the scalp and back for 2 months, with
recent oral ulceration in a 44-year-old person suggests that this is likely to be pemphigus vulgaris. Therefore,
oral prednisolone is the most appropriate empirical treatment until the diagnosis is confirmed by histology
and direct immunofluorescence of the skin biopsy.

Question 180 No Answer Given Incorrect

A 23-year-old man was admitted with a 3-day history of lower cramping abdominal pain and fever. He had
mild diarrhoea, but no blood in his stool. He had not travelled recently.

On examination, he was tender in his right iliac fossa. There were raised, tender, red nodules on his shins.

Investigations:

CT scan of cluster of lymph nodes at appendix suggestive of


abdomen mesenteric adenitis
What is the most likely cause of his symptoms?

Answers

A: Campylobacter jejuni infection


B: Salmonella enteritidis infection
C: tuberculosis
D: ulcerative colitis
E: Yersinia enterocolitica infection

Correct answer: E
Explanation
The history of a nonspecific diarrhoeal disease with right iliac fossa pain would most commonly be due to
campylobacter. However, the finding of a “cluster of lymph nodes at appendix suggestive of mesenteric
adenitis” points more to yersinia infection, which can mimic Crohn’s disease and appendicitis
(pseudoappendicitis).

Question 181 No Answer Given Incorrect


A 78-year-old man presented with a 3-month history of haematuria, increasing tiredness and breathlessness
on exertion. 

On examination, he looked pale.

Investigations:

haemoglobin 78 g/L (130–180)


MCV 108 fL (80–96)
white cell count 2.6 × 109/L (4.0–11.0)
neutrophil
0.9 × 109/L (1.5–7.0)
count
platelet count 78 × 109/L (150–400)
   
nucleated red blood cells, myelocytes and
blood film
metamyelocytes
What is the most likely cause of his abnormal blood film?

Answers

A: aplastic anaemia
B: bone marrow metastases
C: hypothyroidism
D: myelodysplasia
E: vitamin B12 deficiency

Correct answer: B
Explanation
The investigations demonstrate pancytopenia with blood film appearances consistent with bone marrow
infiltration (a leukoerythroblastic film). The other options may all induce pancytopenia but would be
associated with different blood film appearances. Aplastic anaemia and hypothyroidism generally are
associated with a normal blood film, myelodysplasia with abnormal blood cell appearances (e.g.
poikilocytosis) and vitamin B12 deficiency  with the characteristic hypersegmented neutrophils. A further clue
in the stem is the haematuria, which indicates an underlying pathology. 

Question 182 No Answer Given Incorrect

A 24-year-old woman presented with a 4-month history of a non-itchy rash on her trunk. The rash was
originally fawn-coloured and the areas had become white following sun exposure. It had not improved with
clobetasone butyrate cream. She was otherwise well.
On examination, she was suntanned and there was a fine, scaly eruption consisting of discrete, pale macules,
coalescing into large patches.

What is the most appropriate initial treatment?

Answers

A: aqueous cream
B: narrow-band UVB phototherapy
C: nystatin cream
D: oral flucloxacillin
E: oral fluconazole

Correct answer: E
Explanation
The history of fawn-coloured areas on untanned skin which become hypopigmented/white when the rest of
the patient’s skin tans following repeated sun exposure indicates that this patient has pityriasis versicolor,
which is caused by a yeast fungus of the genus malassezia (formerly known as pityrosporum). Treatment is
with azoles (imidazoles such as ketoconazole, or triazoles such as fluconazole), and oral therapy is frequently
used when the rash is extensive.

Question 183 No Answer Given Incorrect

A 55-year-old man presented saying he had coughed up blood on several occasions.

Investigations showed a squamous cell carcinoma of the lung.

What associated finding is a contraindication to surgical resection?

Answers

A: finger clubbing
B: hypercalcaemia
C: hypertrophic pulmonary osteoarthropathy
D: superior vena cava obstruction
E: transfer factor for CO (TLCO) >50% predicted

Correct answer: D
Explanation
The presence of superior vena cava obstruction at presentation is indicative of locally advanced and invasive
disease, and portents inoperability and overall poorer survival outcomes.

Hypercalcaemia could be a paraneoplastic manifestation and does not necessarily preclude curative surgical
resection.
Although predicted post-operative TLCO needs to be formally calculated based on the extent of resection, a
baseline TLCO >50% predicted is a favourable finding.

Question 184 No Answer Given Incorrect

A 70-year-old woman presented after being found collapsed at home.

Examination showed weakness of the lower two-thirds of the left side of her face and a dense left
hemiparesis affecting her arm and leg. She had a fixed gaze palsy to the right affecting both eyes.

Damage to what area of the brain is most likely to have produced the gaze palsy?

Answers

A: left frontal lobe


B: left medulla oblongata
C: left occipital lobe
D: right frontal lobe
E: right occipital lobe

Correct answer: D
Explanation
Left hemiparesis with left upper motor neurone facial nerve palsy, implying a right-sided brain lesion. Fixed
gaze palsy to the right supports this (the eyes ‘look towards’ the side of the lesion). These findings are typical
of a frontal lobe lesion:

Posterior circulation (occipital) stroke would cause ipsilateral cranial nerve signs and contralateral limb signs
(‘crossed deficit’).

Question 185 No Answer Given Incorrect

A 62-year-old woman presented with a 1-hour history of left-sided weakness. She was found to have an
ischaemic stroke. A decision was made to administer alteplase.

What best describes the mechanism of action of alteplase?

Answers

A: acceleration of plasminogen to plasmin conversion


B: adenosine diphosphate (ADP) P2Y12 receptor antagonism
C: cyclooxygenase-1 inhibition
D: factor Xa inhibition
E: vitamin K epoxide reductase inhibition
Correct answer: A
Explanation
Alteplase is a human tissue plasminogen activator. It binds to fibrin-rich clots and cleaves plasminogen to
form plasmin which then degrades the fibrin matrix of the thrombus.

Question 186 No Answer Given Incorrect

A 26-year-old woman with a previous history of deep venous thrombosis was treated with low-molecular-
weight heparin for a further episode proven by a Doppler ultrasound scan. She also reported that she had
had two first-trimester spontaneous abortions.

Investigations:

prothrombin time 13.0 s (11.5–15.5)


activated partial thromboplastin time 50 s (30–40)
   
anticardiolipin IgG antibodies 62 U/mL (<10)
What is the most appropriate treatment?

Answers

A: aspirin
B: aspirin and clopidogrel
C: prednisolone
D: warfarin for 6 months
E: warfarin indefinitely

Correct answer: E
Explanation
This history makes clear that this is a case of recurrent venous thromboembolism in a young patient. There is
also the history of recurrent early pregnancy loss, which would be in keeping with a prothrombotic tendency.
The patient merits indefinite anticoagulation for venous thrombosis, which is answer option E. Aspirin, and
aspirin and clopidogrel, although also indicated as long-term therapy, would not be appropriate for recurrent
venous thrombosis.

Question 187 No Answer Given Incorrect

A 30-year-old woman complained of a hand tremor at a routine transplant clinic review. She had had a
successful renal transplant 3 weeks previously. Her medication included mycophenolate mofetil, tacrolimus,
prednisolone, co-trimoxazole and valganciclovir.

Which drug is most likely to be the cause of her tremor?


Answers

A: co-trimoxazole
B: mycophenolate mofetil
C: prednisolone
D: tacrolimus
E: valganciclovir

Correct answer: D
Explanation
This is a well-known and common adverse effect of tacrolimus, for which patients are counselled
beforehand.

Question 188 No Answer Given Incorrect

In a healthy subject, what is most likely to decrease pulmonary vascular resistance?

Answers

A: endothelin
B: epoprostenol (prostacyclin)
C: hypocapnia
D: hypoxia
E: noradrenaline (norepinephrine)

Correct answer: B
Explanation
Pulmonary vascular resistance (PVR) is mainly generated by arterioles, blood viscosity and lung volume. It is
increased by hypoxia, noradrenaline (norepinephrine) and endothelin. The effects of hypocapnia are variable
(therefore, not the best answer of the five). PVR is decreased by epoprostenol (prostacyclin).

Question 189 No Answer Given Incorrect

A 30-year-old woman presented with a 1-year history of pain in her right hand, spreading to the whole arm
and the scapular and pectoral areas.

On examination, there was no wasting, but there was mild weakness of the right hand and wrist. The right
biceps, supinator and triceps reflexes were all absent. On sensory examination, there was impairment of
pinprick and temperature sensation throughout the right arm.

What is the most likely diagnosis?

Answers
A: cervical spondylosis
B: malignant infiltration of the right brachial plexus
C: multiple sclerosis
D: subacute combined degeneration of the spinal cord
E: syringomyelia

Correct answer: E
Explanation
The combination of lower motor neurone (LMN) signs and spinothalamic loss is suggestive of this (rare)
diagnosis, syringomyelia. Spondylosis is a radiological diagnosis, the story is too long for malignancy, and
multiple sclerosis (MS) would usually lead to upper motor neurone (UMN) not LMN signs (although LMN
signs can rarely occur in MS). Subacute combined degeneration of the spinal cord typically affects the
posterior columns.

Question 190 No Answer Given Incorrect

A 65-year-old man with atrial fibrillation, which was suboptimally controlled by atenolol, was treated with
the addition of oral digoxin 250 micrograms daily. His physician explained that the full effect of this
treatment would not be apparent for at least a week.

On which pharmacokinetic variable did the physician base this explanation?

Answers

A: bioavailability
B: half-life
C: plasma protein binding
D: rate of hepatic metabolism
E: renal clearance

Correct answer: B
Explanation
Digoxin has an average half-life of 36 hours. Steady state digoxin concentration after daily oral dosing is
achieved after 5 half-lives i.e. around 1 week.

Question 191 No Answer Given Incorrect

A 45-year-old man with Down syndrome underwent chromosome analysis to provide information about the
possibility of recurrence in his family.

Chromosome analysis confirmed that his karyotype was 47, XY, +21.

What term best describes this karyotype?


Answers

A: aneuploid
B: diploid
C: haploid
D: monoploid
E: triploid

Correct answer: A
Explanation
This karyotype describes a male with 23 pairs of chromosomes plus an additional chromosome 21.
Aneuploidy is the term used to describe the presence of an abnormal number of chromosomes (either less
than 46 or more) but not abnormal numbers of complete chromosome sets. The other options refer to
abnormal number of complete sets of chromosomes – fewer or more than 2

Question 192 No Answer Given Incorrect

A 60-year-old man was admitted with seizures and altered mental status. He was treated with intravenous
aciclovir for suspected viral encephalitis. The product literature advised maintaining adequate hydration to
avoid renal injury.

What is the mechanism of renal injury with aciclovir?

Answers

A: acute tubular necrosis


B: crystalluria
C: glomerulonephropathy
D: interstitial nephritis
E: renal ischaemia

Correct answer: B
Explanation
Aciclovir is renally eliminated. In the presence of dehydration, aciclovir can precipitate as crystals in the kidney
tubules causing acute kidney injury.

Question 193 No Answer Given Incorrect

A 70-year-old woman presented following the sudden onset of painless diplopia.

On examination, movements of her left eye were limited in all directions. There was a mild left-sided ptosis.
Her left pupil was larger than her right and reacted poorly to light (direct and consensual). There was loss of
pinprick sensation on her right forehead.
What is the most likely site of the lesion?

Answers

A: left cavernous sinus


B: left cerebellopontine angle
C: left medulla
D: left midbrain
E: left pons

Correct answer: A
Explanation
The examination suggests a lesion involving III, IV, V(1), and VI, and the only location for such a lesion would
be cavernous sinus; the other distractors could not explain such a lesion.

Question 194 No Answer Given Incorrect

A 23-year-old woman was referred for further investigation of primary infertility. Her menarche had occurred
at the age of 13 and her menstrual cycle varied in length from 24 to 76 days.

On examination, her BMI was 33.0 kg/m2 (18.0–25.0). She had mild facial hirsutism.

Investigations:

serum 17-hydroxyprogesterone 8 nmol/L (1–10)


serum oestradiol 225 pmol/L (200–400)
serum follicle-stimulating hormone 4.2 U/L (2.5–10.0)
serum luteinising hormone 8.8 U/L (2.5–10.0)
serum prolactin 525 mU/L (100–360)
serum thyroid-stimulating hormone 4.8 mU/L (0.4–5.0)
What is the most likely diagnosis?

Answers

A: endometriosis
B: non-classical congenital adrenal hyperplasia
C: polycystic ovary syndrome
D: prolactinoma
E: subclinical hypothyroidism

Correct answer: C
Explanation
The clinical scenario of irregular menstruation, infertility and hirsutism, indicative of androgen excess in a
young woman, is due to polycystic ovary syndrome.

Question 195 No Answer Given Incorrect

A 17-year-old girl of Indo-Asian descent presented with a typical rash of erythema nodosum over her shins.
She felt otherwise well. She had been born in the UK and had no previous medical problems.

A chest X-ray showed a very abnormal mediastinal silhouette.

Investigations: 

serum lactate
370 U/L (10–250)
dehydrogenase
serum
angiotensin-
78 U/L (25–82)
converting
enzyme
   
serum α-
1 kU/L (<10)
fetoprotein
serum β-
human
<1 U/L (<5)
chorionic
gonadotropin
   
12-cm-diameter, complex soft-tissue lesion in anterior
mediastinum, encasing great vessels in superior
CT scan of
mediastinum; enlarged left para-cardiac and hilar lymph
thorax
nodes up to 11 mm in diameter; no further abnormality
in thorax or upper abdomen
What is the most likely diagnosis?

Answers

A: Hodgkin disease
B: sarcoidosis
C: teratoma
D: thymoma
E: tuberculosis

Correct answer: A
Explanation
The syndrome described is an anterior mediastinal mass in an individual in the second decade of life.
Erythema nodosum, the other presenting feature described, is associated with Hodgkin disease, sarcoidosis
and tuberculosis.

As a diagnostic test for sarcoidosis, measurement of serum angiotensin-converting enzyme (ACE) levels lacks
sensitivity and specificity. Up to 40% of cases of sarcoidosis will have normal serum ACE levels.

The normal serum α-fetoprotein and serum β-human chorionic gonadotropin levels make teratoma unlikely.

Scan suggests a malignant condition, and coupled with the serum lactate dehydrogenase elevation, make
Hodgkin disease the most likely diagnosis.

Question 196 No Answer Given Incorrect

A 16-year-old boy presented with a 9-month history of a right-sided neck swelling, which had steadily
increased in size over the previous 3 months but had remained painless. He had no other symptoms.

On examination, he had enlarged pre-auricular and jugulodigastric lymph nodes and there was no other
abnormality. 

Investigations:

chest X-ray normal


What next investigation would be most useful in making a diagnosis?

Answers

A: bone marrow examination


B: CT scan of head and neck
C: excision biopsy of cervical lymph node
D: fine-needle aspiration of cervical lymph node
E: Mantoux skin test

Correct answer: C
Explanation
This is longstanding painless and progressive neck adenopathy in an adolescent. The most likely diagnosis is
classical Hodgkin lymphoma. The most appropriate diagnostic investigation is excision biopsy. Fine-needle
aspiration is likely to yield insufficient material for diagnostic purposes in Hodgkin lymphoma and, therefore,
is likely to be unhelpful and delay diagnosis. Bone marrow examination and CT scan of head and neck are
investigations reserved for staging rather than diagnosis. Mantoux skin test is not likely to be helpful in this
setting as Hodgkin lymphoma is the most likely diagnosis.
Question 197 No Answer Given Incorrect

An 80-year-old man underwent carotid endarterectomy after an ischaemic stroke.

Investigations:

carotid plaque histology presence of foam cells within the intima


In atherosclerosis, what is the origin of foam cells?

Answers

A: B lymphocytes
B: basophils
C: endothelial cells
D: monocytes
E: neutrophils

Correct answer: D
Explanation
Foam cells are the fat-laden M2 macrophages that serve as the hallmark of early stage atherosclerotic lesion
formation. Foam cell formation is triggered by a number of factors including the uncontrolled uptake of
modified low-density lipoprotein (LDL), the upregulation of cholesterol esterification and the impairment of
mechanisms associated with cholesterol release. They are formed when circulating monocyte-derived cells are
recruited to the atherosclerotic lesion site or fat deposits in the blood vessel walls.

Question 198 No Answer Given Incorrect

A 24-year-old woman presented with recurrent joint pains. A range of investigations, including a test for
autoimmune serology, was performed. Antinuclear antibodies were detected, but the isotype was not
indicated.

What antinuclear antibody isotype is most likely to indicate a diagnosis of systemic lupus erythematosus?

Answers

A: IgA
B: IgD
C: IgE
D: IgG
E: IgM

Correct answer: D
Explanation
IgG antinuclear antibodies are associated with systemic lupus erythematosus.

Question 199 No Answer Given Incorrect

A 20-year-old woman presented with fever, headache and a non-blanching rash of 12 hours’ duration.

She was found to have meningococcal meningitis and septicaemia, and started treatment with appropriate
therapy.

The consultant in Public Health Medicine was notified and advised prophylactic antibiotics for her close
contacts. She shared a house with three friends, who were all well.

What is the most appropriate prophylaxis for her housemates?

Answers

A: cefuroxime
B: chloramphenicol
C: ciprofloxacin
D: doxycycline
E: phenoxymethylpenicillin

Correct answer: C
Explanation
This is in keeping with guidance from Public Health England.

Question 200 No Answer Given Incorrect

A 28-year-old man presented with a 1-month history of fever, joint pains and a non-itchy rash that tended to
occur in the mornings.

On examination, his temperature was 39.0°C. He had swollen and warm ankles and wrists, a maculopapular
rash on his trunk and legs, and splenomegaly.

Investigations:

haemoglobin 110 g/L (130–180)


white cell count 21.0 × 109/L (4.0–11.0)
neutrophil count 19.5 × 109/L (1.5–7.0)
ESR 78 mm/1st h (<15)
serum ferritin 4500 µg/L (15–300)
   
chest X-ray normal
What is the most likely diagnosis?

Answers

A: adult-onset Still’s disease


B: haemochromatosis
C: Lyme disease
D: lymphoma
E: sarcoidosis

Correct answer: A
Explanation
The correct answer is adult-onset Still’s disease. Still’s disease is a rare disease and more often seen in children
than adults. It is a systemic inflammatory illness. Clinical features include fever, joint pain/inflammation and
evanescent macular rash. A characteristic lab feature is very high levels of ferritin; additional lab features
include anaemia, leukocytosis and high acute-phase reactants. This scenario showcases all these features in
an adult patient. Haemochromatosis would not cause systemic inflammatory illness but can lead to
arthralgia, low-grade localised joint inflammation and chondrocalcinosis. Lyme disease is an infection spread
by tick bite; the characteristic rash is erythema migrans. Sarcoidosis commonly presents as erythema
nodosum, arthralgia, periarthritis (mainly at ankles) and hilar/mediastinal lymphadenopathy. Lymphoma is less
likely given the fact that the chest X-ray is normal and the serum ferritin is very high.

Useful links
Cookies policy
Terms & conditions
Document library
Sitemap
Privacy policy

@MRCPUK

You might also like